You are on page 1of 124

C

h apter
ontents
PHYSICS
CBSE–XII
TERM – I
Syllabus I & II

1. Electric Charges & Fields 01-16

2. Electrostatic Potential & Capacitance 17-34

3. Current Electricity 35-56

4. Moving Charges and Magnetism 57-76

5. Magnetism and Matter 77-86

6. Electromagnetic Induction 87-102

7. Alternating Current 103-120


Physics I

PHYSICS
CLASS XII (2021 - 22)
SYLLABUS
TERM – I
Units Topics No. of Periods Marks
Unit - I Electrostatics 23
Chapter–1: Electric Charges and Fields
Chapter–2: Electrostatic Potential and Capacitance
Unit – II Current Electricity 15
17
Chapter–3: Current Electricity
Unit – III Magnetic Effects of Current and Magnetism 16
Chapter–4: Moving Charges and Magnetism
Chapter–5: Magnetism and Matter
Unit – IV Electromagnetic Induction and Alternating Current 19
Chapter–6: Electromagnetic Induction 18
Chapter 7: Alternating current
Total 73 35

UNIT I : ELECTROSTATICS 23 Periods


Chapter–1: Electric Charges and Fields
Electric Charges; Conservation of charge, Coulomb's law-force between two-point charges,
forces between multiple charges; superposition principle and continuous charge
distribution. Electric field, electric field due to a point charge, electric field lines, electric
dipole, electric field due to a dipole, torque on a dipole in uniform electric field. Electric
flux, statement of Gauss's theorem and its applications to find field due to infinitely long
straight wire, uniformly charged infinite plane sheet
Chapter–2: Electrostatic Potential and Capacitance
Electric potential, potential difference, electric potential due to a point charge, a dipole and
system of charges; equipotential surfaces, electrical potential energy of a system of two-
point charges and of electric dipole in an electrostatic field. Conductors and insulators, free
charges and bound charges inside a conductor. Dielectrics and electric polarisation,
capacitors and capacitance, combination of capacitors in series and in parallel, capacitance
of a parallel plate capacitor with and without dielectric medium between the plates, energy
stored in a capacitor.

E
II Physics
UNIT II : CURRENT ELECTRICITY 15 Periods
Chapter–3: Current Electricity
Electric current, flow of electric charges in a metallic conductor, drift velocity, mobility
and their relation with electric current; Ohm's law, electrical resistance, V-I characteristics
(linear and non-linear), electrical energy and power, electrical resistivity and conductivity;
temperature dependence of resistance. Internal resistance of a cell, potential difference and
emf of a cell, combination of cells in series and in parallel, Kirchhoff's laws and simple
applications, Wheatstone bridge, metre bridge(qualitative ideas only). Potentiometer -
principle and its applications to measure potential difference and for comparing EMF of
two cells; measurement of internal resistance of a cell (qualitative ideas only)

UNIT III: MAGNETIC EFFECTS OF CURRENT AND MAGNETISM 16 Periods


Chapter–4: Moving Charges and Magnetism
Concept of magnetic field, Oersted's experiment. Biot - Savart law and its application to
current carrying circular loop. Ampere's law and its applications to infinitely long straight
wire. Straight and toroidal solenoids (only qualitative treatment), force on a moving charge
in uniform magnetic and electric fields. Force on a current-carrying conductor in a uniform
magnetic field, force between two parallel current-carrying conductors-definition of
ampere, torque experienced by a current loop in uniform magnetic field; moving coil
galvanometer-its current sensitivity and conversion to ammeter and voltmeter.

Chapter–5: Magnetism and Matter


Current loop as a magnetic dipole and its magnetic dipole moment, magnetic dipole
moment of a revolving electron, bar magnet as an equivalent solenoid, magnetic field lines;
earth's magnetic field and magnetic elements.

UNIT IV: ELECTROMAGNETIC INDUCTION AND ALTERNATING CURRENTS 19 Periods


Chapter–6: Electromagnetic Induction
Electromagnetic induction; Faraday's laws, induced EMF and current; Lenz's Law, Eddy
currents. Self and mutual induction.

Chapter–7: Alternating Current


Alternating currents, peak and RMS value of alternating current/voltage; reactance and
impedance; LC oscillations (qualitative treatment only), LCR series circuit, resonance;
power in AC circuits. AC generator and transformer.

E
Physics 1
UNIT – I : ELECTROSTATICS
CHAPTER – 1 : ELECTRIC CHARGES & FIELDS

1. ELECTRIC CHARGE
Charge is a property of substance due to which it shows electric and magnetic effects.
Kinds of charges : positive and negative.
S.I. unit of charge is Coulomb (C).
Basic property of charge : (a) Additivity (b) Conservation (c) Quantisation
Mode of charging : (a) charging by friction (b) charging by conduction (c) charging by
induction
Note : Electric charge is invariant because value of electric charge does not depend on frame of
reference.

2. COULOMB'S LAW
The mutual electrostatic force between two point charges (q1 and q2) is proportional to the
product (q1q2) and inversely proportional to the square of the distance (r2) separating them. The
force acts along the line joining two charges.
1 q1q2
Force in free space (air / vacuum) F  ˆr
40 r 2
q1 q2
r
1 q1q2
Force in any medium F rˆ
4 r 2
where     0  r 
r = relative permittivity, = permittivity of medium, 
 0 = permittivity of free space (air / vacuum) = 8.8 × 10–12 C2N–1m–2
NOTE :The Law is applicable only for static and point charges. Moving charges may result in
magnetic interaction and if charges are spread on bodies then induction may change the charge
distribution.

3. ELECTRIC FIELD / ELECTRIC INTENSITY / ELECTRIC FIELD STRENGTH ( )


It is defined as the net force on unit positive charge due to all other charges. It is vector quantity.
F
E unit is N/C or V/m.
q

kq
(a) Electric field due to point charge E  ˆr
r2

E
2 Physics

Null point for two charges : 

If |Q1| > |Q2| Null point near Q2 (Smaller charge) 


Q1
x r (distance of null point from Q1)
Q1  Q2
(+) for like charges; (–) for unlike charges

4. ELECTRIC FLUX : 
      E.dA
N 2
SI unit = ·m
C
(i) For uniform electric field;   E.A  EAcos  where  = angle between E & area vector
 A  . Flux is contributed only due to the component of electric field which is perpendicular
to the plane.
(ii) If E is not uniform throughout the area A , then    E.dA

(iii) dA represent area vector normal to the surface and pointing outwards from a closed
surface.

5. GAUSS’S LAW
1
It states that flux of electric field through any closed surface S is times the total charge
0
enclosed by S.
q
 E.ds  0
(Applicable only to closed surface)

qen
   Ed A   
0
where qen= net charge enclosed by the closed surface. The above closed surface is called gaussian
surface. 
  does not depend on the :
(i) Shape and size of the gaussian surface
(ii) The charges located outside the gaussian surface.
(iii) Location of chage inside the gaussian surface.
Application of Gauss's Law

E
Physics 3
(i) Field due to an infinitely long straight uniformly charged wire :

E nˆ
20 r
+
+ Cylindrical Gaussian
+
+
+ surface
+
+
+
+
+
+
P r +
+
+
dA E + |E|
+
+
+
+
+
+
+
+
+ O
+ r

(ii) Field due to uniformly charged infinite plane non conducting sheet.

E nˆ
2 0

++
++++ ^
 + +
++ +
+ ++
++

6. ELECTROSTATIC FIELD LINES


An electrostatic field line is a curve drawn in such a way that the tangent at each point on the
curve gives the direction of electric field at that point.

Field lines of electrostatic field have following properties :


(i) Imaginary lines
(ii) Never intersect each other
(iii) Electrostatic field lines never forms closed loops
(iv) Field lines ends or starts normally at the surface of a conductor.
(v) If there is no electric field there will be no field lines.
(vi) Number of electric field lines per unit area normal to the area at a point represents
magnitude of intensity, crowded lines represent strong field while distant lines weak field.

E
4 Physics
7. ELECTRIC DIPOLE
If equal and opposite point charges are placed at very small separation then system is known as
dipole.

Electric dipole moment ( )


It is defined as the product of magnitude of either charge and distance between them.
·2a
Electric dipole moment : (i) It is a vector quantity, (ii) Direction is from –ve to +ve charge.

Electric dipole in uniform electric field


1. Torque   p  E or   pEsin 
2. Work done in rotation to dipole from 1 to 2 angle in external electric field.
W = pE(cos1–cos2)
3. Potential energy = –pE cos

Note :
• In uniform field, force on a dipole = 0, torque may or may not be zero.
• In general in non-uniform field, force on a dipole  0 and torque may or may not be zero
dE
• In non-uniform electric field  Fe  p.
dr
• When P is parallel to E then the dipole is in stable equilibrium
• When P is antiparallel to E then the dipole is in unstable equilibrium

Electric field and potential due to dipole


S.No. Electric field Potential
2kp kp
1. at axial
r3 r2
kp
2. at equatorial 0
r3
kp kp cos 
3. at general position 3
1  3cos2 
r r2

E
Physics 5
PRACTICE QUESTIONS
CASE STUDY BASED QUESTIONS
Type -1
Guass's law and its significance
Gauss's law and coulomb's law, although expressed in different forms, are equivalent ways of
describing the relation between charge and electric field in static conditions. Gauss's law is
0 = qencl, when qencl is the net charge inside an imaginary closed surface called Gaussian
Surface    E.dA gives the electric flux through the Gaussian surface. The two equations hold
only when the net charge is in vacuum or air.

+Q

Gaussian spherical surfaces


1. If there is only one type of charge in the universe, then
( E  Electric field, ds  Area Vector)
(1)  E.ds  O on any surface
(2)  E.ds could not be defined
(3)  E.ds   if charge is inside
q
(4)  E.ds  O if charge is outside,  E.ds   0
if charge is inside

2. What is the nature of Gaussian surface involved in Gauss law of electrostatic?


(1) Magnetic (2) Scalar (3) Vector (4) Electrical
3. A charge 10C is placed at the centre of a hemisphere of radius R = 10 cm as shown.
The electric flux through the hemisphere (in MKS units) is____
+10
C

(1) 20 × 105 (2) 10.5 × 105 (3) 5.5 × 105 (4) 2 × 105
4. The electric flux through a closed surface area S enclosing charge Q is . If the surface area is
doubled, then the flux is ______.
(1) 2 (2) /2 (3) /4 (4) 
5. A Gaussian surface encloses a dipole. The electric flux through this surface is______
q 2q q
(1) (2) (3) (4) zero
0 0 20

E
6 Physics
Type 2 : Assertion and Reason :-
Question 6 to 11:-Note:- In the following questions an Assertion (A) is followed by a corresponding
Reason (R). Use the following keys to choose the appropriate answer.
(1) Both (A) and (R) are true, (R) is the correct explanation of (A).
(2) Both (A) and (R) are true, (R) is not the correct explanation of (A).
(3) (A) is true; (R) is false.
(4) (A) is false; (R) is false.
6. Assertion : Range of coulomb force is infinite.
Reason : Coulomb force acts between two charged particles.
7. Assertion : A metallic shield in form of a hollow shell may be built to block an electric field.
Reason : In a hollow spherical shield, the electric field inside it is zero at every point.
8. Assertion : If a proton and an electron are placed in the same uniform electric field. They
experience different acceleration.
Reason : Electric force on a test charge is independent of its mass.
9. Assertion : Charge is invariant.
Reason : Charge does not depends on speed of frame of reference.
10. Assertion : Electric flux density is independent of medium.
Reason : It depends only on distance and charge.
11. Assertion : Charge is quantized.
Reason : Charge, which is less than 1 C is not possible.

Type 3 : Multiple choice question (one correct answer) (Questions 12 to 21):-


12. Force of attraction between two point charges Q and –Q separated by d metre is Fe . When these

charges are placed on two identical spheres of radius R  0.3d whose centres are d metre apart,
the force of attraction between them is :
(1) Greater than Fe (2) Equal to Fe (3) Less than Fe (4) Less than Fe

13. Two point charges 3C and 8C repel each other with a force of 40N . If a charge of 5C
is added to each of them, then the force between them will becomes :
(1) 10N (Attractive) (2) 10N (Repulsive) (3) 20N (Repulsive) (4) 20N (Attractive)
14. The negative electric flux indicates that The net flux through the surface is :
(1) outward (2) inward
(3) may be outward or inward (4) neither outward not inward

E
Physics 7
15. Three charges are placed at the vertices of an equilateral triangle of side ‘a’ as shown in the
following figure. The force experienced by the charge placed at the vertex A in a direction
normal to BC is :
A
+Q

–Q +Q
B a C

(1) Q2 / (40a 2 ) (2) Q / (40a 2 ) (3) Zero (4) Q2 / (20a 2 )

16. The insulation property of air breaks down at E  3 106 volt/metre. The maximum charge that
can be given to a sphere of diameter 5m is approximately (in coulombs) :

(1) 2 102 (2) 2 103 (3) 2 104 (4) 2 105


17. The S.I. unit of electric flux is :
(1) volt/m2 (2) newton–metre (3) volt–metre (4) newton m2 /
(coulomb)2
18. The force between two charges is 120 N. If the distance between the charge is doubled. The force
will be :
(1) 60 N (2) 30 N (3) 40 N (4) 15 N
19. The direction of electric field due to positive charge is :
(1) Away from the charge (2) Towards the charge
(3) Both (1) & (2) (4) None of the above
20. The charge on 500 cc of water due to protons will be :

(2) 2.67 10 C (4) 1.67 10 C


7 23
(1) 6.0 1027 C (3) 6 1023 C
21. Two charges 4e and e are at a distance x apart. At what distance, a charge q must be placed
from charge e so that it is in equilibrium :
(1) x/2 (2) 2x/3 (3) x/3 (4) x/6

ANSWER KEY
1. (4) 2. (3) 3. (3) 4. (4) 5. (4) 6. (2) 7. (1) 8. (2)
9. (1) 10. (4) 11. (3) 12. (1) 13. (1) 14. (2) 15. (3) 16 (2)
17. (3) 18. (2) 19. (1) 20. (2) 21. (3)

E
8 Physics
ASSIGNMENT-1
(Electric charges and Coulomb’s law)
1. There are two metallic spheres of same radii but one is solid and the other is hollow, then :
(1) Solid sphere can be given more charge
(2) Hollow sphere can be given more charge
(3) They can be charged equally (maximum)
(4) None of the above
2. One metallic sphere A is given positive charge whereas another identical metallic sphere B of
exactly same mass as of A is given equal amount of negative charge. Then :
(1) Mass of A and mass of B still remain equal
(2) Mass of A increases
(3) Mass of B decreases
(4) Mass of B increases
3. Two charges are at a distance ‘d’ apart. If a copper plate (conducting medium) of thickness d is
placed between them, the effective force will be :
(1) 2F (2) F/2 (3) 0 (4)2F
4. Two charges of equal magnitudes and at a distance r exert a force F on each other. If the charges
are halved and distance between them is doubled, then the new force acting on each charge is :

(1) F/8 (2) F/4 (3) 4F (4) F/16


5. Electric field lines due to charged particles are :
(1) Always straight (2) Always curved (3) Sometimes curved (4) None of the above

ASSIGNMENT-2
(Electric Field and Electric Field Lines)
6. ABC is an equilateral triangle. Charges +q are placed at each corner. The electric intensity at O
will be :
+q
A

r r
O
+q +q
B C

1 q 1 q 1 3q
(1) (2) (3) Zero (4)
40 r 2 40 r 40 r 2
7. The magnitude of electric field intensity E is such that, an electron placed in it would experience
an electrical force equal to its weight is given by :
mg e e2
(1) mge (2) (3) (4) 2 g
e mg m

E
Physics 9
8. A charge particle is free to move in an electric field. It will travel :
(1) Always along a line of force
(2) Along a line of force, if its initial velocity is zero
(3) Along a line of force, if it has some initial velocity in the direction of an acute angle with the
line of force
(4) None of the above
9. An uncharged sphere of metal is placed in between two charged plates as shown. The lines of
force look like :
++ ++ ++ + ++ ++ ++ +
++ ++ ++ + ++ ++ ++ +

– – – – – – – – – – – – – – – – – – – – – – – – – – – –
A B C D

(1) A (2) B (3) C (4) D


10. The distance between a proton and electron both having a charge 1.6 1019 coulomb , of a
hydrogen atom is 1010 metre . The value of intensity of electric field produced on electron due to
proton will be :
(1) 2.304 1010 N / C (2) 14.4 V / m (3) 16 V / m (4) 1.44 1011 N / C

ASSIGNMENT-3
(Electric Dipole)
11. The electric field due to a dipole at a distance r on its axis is :
(1) Directly proportional to r3 (2) Inversely proportional to r3
(3) Directly proportional to r2 (4) Inversely proportional to r2
12. An electric dipole consisting of two opposite charges of 2 106 C each separated by a distance
of 3cm is placed in an electric field of 2  105 N/C. The maximum torque on the dipole will be :
(1) 12 101 Nm (2) 12 103 Nm (3) 24 101 Nm (4) 24 103 Nm
13. An electric dipole is kept in non-uniform electric field. It experiences :
(1) A force and a torque (2) A force but not a torque
(3) A torque but not a force (4) Neither a force nor a torque
14. The value of dielectric constant for vacuum is taken as :
(1) zero (2) 1 (3) 4 (4) 10
15. The electric intensity due to a dipole of length 10 cm and having a charge of 500 C, at a point
on the axis at a distance 20 cm from one of the charges in air, is :
(1) 6.25 107 N/C (2) 9.28 107 N/C (3) 13.11111 N/C (4) 20.5 107 N/C

E
10 Physics
ASSIGNMENT-4
(Electric Flux and Gauss’s Law)

16. A cube of side l is placed in a uniform field E, where E  Eˆi . The net electric flux through the
cube is :

(1) Zero (2) l 2 E (3) 4l 2 E (4) 6l 2 E


17. It is not convenient to use a spherical Gaussian surface to find the electric field due to an
infinitely long strength uniformly charged wire using Gauss’s theorem because :
(1) Gauss’s law fails in this case
(2) It does not have spherical symmetry
(3) Coulomb’s law is more fundamental than Gauss’s law
(4) Spherical Gaussian surface will alter the dipole moment
18. A charge q is placed at the centre of the open end of cylindrical vessel. The flux of the electric
field through the surface of the vessel is:
q q 2q
(1) Zero (2) (3) (4)
0 20 0

19. If the electric flux entering and leaving an enclosed surface respectively is 1 and 2 the electric

charge inside the surface will be :


(1) (1  2 )0 (2) (2  1 )0 (3) (1  2 ) / 0 (4) (2  1 ) / 0

20. Gauss’s law should be invalid if


(1) There were magnetic monopoles
(2) The inverse square law were not exactly true
(3) The velocity of light were not a universal constant
(4) None of these

ANSWER KEY
1. (3) 2. (4) 3. (3) 4. (4) 5. (2) 6. (3) 7. (2) 8. (2)
9. (3) 10. (4) 11. (2) 12. (2) 13. (1) 14. (2) 15. (1) 16 (1)
17. (2) 18. (3) 19. (2) 20. (2)

E
Physics 11

EXERCISE-1
(Electric charges and Coulomb’s law)
1. A body can be negatively charged by :
(1) Giving excess of electrons to it (2) Removing some electrons from it
(3) Giving some protons to it (4) Removing some neutrons from it
2. When the distance between the charged particles is halved, the force between them becomes :
(1) One-fourth (2) Half (3) Double (4) Four times
3. Dielectric constant for metal is :
(1) Zero (2) Infinite (3) 1 (4) Greater than 1
4. There are two charges +1 microcoulombs and +5 microcoulombs. The ratio of the forces acting
on them will be :
(1) 1 : 5 (2) 1 : 1 (3) 5 : 1 (4) 1 : 25
5. When 1019 electrons are removed from a neutral metal plate, the electric charge on it is :
(1) – 1.6 C (2) + 1.6 C (3) 10+19 C (4) 10–19 C

(Electric Field and Electric Field Lines)


6. The electric charge in uniform motion produces :
(1) An electric field only (2) A magnetic field only
(3) Both electric and magnetic field (4) Neither electric nor magnetic field
7. Electric lines of force about negative point charge are :
(1) Circular, anticlockwise (2) Circular, clockwise
(3) Radial, inward (4) Radial, outward
8. Electric field strength due to a point charge of 5C at a distance of 80 cm from the charge is :
(2) 7  10 N/C
4
(1) 8  104 N/C (3) 5  104 N/C (4) 4  104 N/C
9. An electron having charge ‘e’ and mass ‘m’ is moving in a uniform electric field E. Its
acceleration will be :
e2 E2 e eE mE
(1) (2) (3) (4)
m m m e
10. The unit of electric field is not equivalent to :
(1) N / C (2) J / C (3) V / m (4) J / C  m

(Electric Dipole)
11. The torque acting on a dipole of moment P in an electric field E is :
(1) P  E (2) P  E
(3) Zero (4) E  P
12. The electric field at a point on equatorial line of a dipole and direction of the dipole moment :
(1) Will be parallel (2) Will be in opposite direction
(3) Will be perpendicular (4) Are not related

E
12 Physics
13. An electric dipole in a uniform electric field experiences (When it is placed at an angle  with the
field) :
(1) Force and torque both (2) Force but no torque
(3) Torque but no force (4) No force and no torque
14. The ratio of electric fields on the axis and at equator of an electric dipole will be :
(1) 1 : 1 (2) 2 : 1 (3) 4 : 1 (4) None of these
15. A region surrounding a stationary electric dipoles has :
(1) Magnetic field only (2) Electric field only
(3) Both electric and magnetic fields (4) No electric and magnetic fields

(Electric Flux and Gauss’s Law)


16. Total electric flux coming out of a unit positive charge put in air is :
(1)  0 (2)  01 (3) (40 )1 (4) 40
17. The S.I. unit of electric flux is :
(1) Weber (2) Newton per coulomb
(3) Volt  metre (4) Joule per coulomb
18. A point charge +q is placed at the centre of a cube of side L. The electric flux emerging from the
cube is :
2
q 6qL q
(1) (2) Zero (3) (4) 2
0 0 6L 0
19. According to Gauss’ Theorem, electric field of an infinitely long straight wire is proportional to :
1 1 1
(1) r (2) 2 (3) 3 (4)
r r r
20. q1, q2, q3 and q4 are point charges located at points as shown in the figure and S is a spherical
Gaussian surface of radius R. Which of the following is true according to the Gauss’s law?
S

q1 R
q4
q2 q3

q1  q2  q3 (q1  q2  q3 )
(1)  (E
s 1  E2  E3 ).dA 
20
(2)  (E
s 1  E2  E3 ).dA 
0
(q1  q 2  q3  q 4 )
(3)  (E
s 1  E2  E3 ).dA 
0
(4) None of the above

ANSWER KEY
1. (1) 2. (4) 3. (2) 4. (2) 5. (2) 6. (3) 7. (3) 8. (2)
9. (3) 10. (2) 11. (2) 12. (2) 13. (3) 14. (2) 15. (2) 16 (2)
17. (3) 18. (1) 19. (4) 20. (2)

E
Physics 13
CASE STUDY BASED QUESTIONS

Torque on a Dipole in a Uniform Electric field

When electric dipole is placed in uniform electric field, its two charges experience equal and
opposite forces, which cancel each other and hence net force on electric dipole in uniform
electric field is zero. However these force are not collinear, so they give rise to some torque on
the dipole. Since net force on electric dipole in uniform electric field is zero. So no work is done
in moving the electric dipole in uniform electric field. However some work is done in rotating the
dipole against the torque acting on it.

+q A


–q 2asin

B 2acos

1. The dipole moment of a dipole in a uniform external field:


E is P . Then the torque  acting on the dipole is:

(1)   P  E (2)   P.E (3)   2 P  E  (4)   P  E 


2. An electric dipole consists of two opposite charges, each of magnitude 1.0 C separated by a
distance of 2.0 cm. The dipole is placed in an external field of 105 NC–1. The maximum torque on
the dipole is_____.
(1) 0.2 × 10–3 Nm (2) 1 × 10–3 Nm (3) 2 × 10–3 Nm (4) 4 × 10–3 Nm
3. Torque on a dipole in uniform electric field is minimum when  is equal to ______
(1) 0° (2) 90° (3) 180° (4) Both (1) and (3)
4. When an electric dipole is held at an angle in a Uniform electric field, the net force F and torque
 on the dipole are
(1) F = 0,  = 0 (2) F  0,   0 (3) F = 0,   0 (4) F  0,  = 0
5. An electric dipole of moment P is placed in an electric field of intensity E. The dipole acquires a
position such that the axis of the dipole makes an angle  with the direction of the field.
Assuming that the potential energy of the dipole to be zero when  = 90°, the torque and the
potential energy of the dipole will respectively be ______.
(1) pE sin  – pE cos (2) pE sin  – 2pE cos 
 (3) pE sin  – 2pE cos (4) pE cos – pE sin 


ANSWER KEY
1. (1) 2. (3) 3. (4) 4. (3) 5. (1)

E
14 Physics
ASSERTION AND REASON QUESTIONS
Directions (Q. Nos. 1-8)
In the following questions an Assertion (A) is followed by a corresponding Reason (R). Use the
following keys to choose the appropriate answer.
(1) Both (A) and (R) are true, (R) is the correct explanation of (A).
(2) Both (A) and (R) are true, (R) is not the correct explanation of (A).
(3) (A) is true; (R) is false.
(4) (A) is false; (R) is false.
1. Assertion (A) : If there exists coulomb's attraction between two bodies, both of them may not be
charged.
Reason (R): Accordingly coulomb's law two bodies having opposite charged attract each
other.
2. Assertion (A) : No two electric line of force can intersect each other.
Reason (R): Tangent at any point of electric line of force gives the direction of electric field.
3. Assertion (A) : As force is a vector quantity, hence electric field intensity is also a vector
quantity.
Reason (R): The unit of electric field intensity is newton per coulomb.
4. Assertion : The surface charge densities of two spherical conductors of different radii are equal.
Then the electric field intensities near their surface are also equal.
Reason : Surface charge density is equal to charge per unit area.
5. Assertion : Three equal charges are situated on a circle of radius r such that they form on
equilateral triangle, then the electric field intensity at the centre is zero.
Reason : The force on unit positive charge at the centre, due to the three equal charges are
represented by the three sides of a triangle taken in the same order. Therefore, electric field
intensity at centre is zero.
6. Assertion : On going away from a point charge or a small electric dipole, electric field
decreases at the same rate in both the cases.
Reason : Electric field is inversely proportional to square of distance from the charge or an
electric dipole.
7. Assertion : The whole charge of a conductor cannot be transferred to another isolated conductor.
Reason : The total transfer of charge from one to another is not possible.
8. Assertion : A point charge is brought in an electric field. The field at a nearby point will
increase, whatever be the nature of the charge.
Reason : The electric field is independent of the nature of charge.

ANSWER KEY
1. (2) 2. (1) 3. (2) 4. (2) 5. (1) 6. (4) 7. (3) 8. (4)

E
Physics 15

EXERCISE-2
1. A total charge Q is broken in two parts Q1 and Q2 and they are placed at a distance R from each
other. The maximum force of repulsion between them will occur, when :
Q Q Q 2Q
(1) Q2  , Q1  Q  (2) Q2  , Q1  Q 
R R 4 3
Q 3Q Q Q
(3) Q2  , Q1  (4) Q1  , Q2 
4 4 2 2
2. A charge Q is divided into two parts of q and Q – q. If the coulomb repulsion between them when
Q
they are separated is to be maximum, the ratio of should be :
q
(1) 2 (2) 1/2 (3) 4 (4) 1/4
3. Two point charges +9e and +e are at 16 cm away from each other. Where should another charge
q be placed between them so that the system remains in equilibrium :
(1) 24 cm from +9e (2) 12 cm from +9e (3) 24 cm from +e (4) 12 cm from +e
4. Four charges are placed on corners of a square as shown in figure having side of 5 cm. If Q is
one microcoulomb, then electric field intensity at centre will be :
Q – 2Q

–Q + 2Q

(1) 1.02 10 N / C upwards (2) 2.04 10 N / C downwards


7 7

(3) 2.04 10 N / C upwards (4) 1.02 10 N / C downwards


7 7

5. Electric charges q,q,  2q are placed at the corners of an equilateral triangle ABC of side l. The
magnitude of electric dipole moment of the system is :
(1) ql (2) 2ql (3) 3ql (4) 4ql
6. The electric intensity due to a dipole of length 10 cm and having a charge of 500C , at a point
on the axis at a distance 20 cm from one of the charges in air, is :
(1) 6.25 10 N/C (2) 9.28 10 N/C (3) 13.111 N/C (4) 20.5 10 N/C
7 7 11 7

7. The electric flux for Gaussian surface A that enclose the charged particles in free space is :
(given q1 = –14 nC, q2 = 78.85 nC, q3 = – 56 nC)
Gaussian
q3 surface A
q1
q2 Gaussian
surface B

(1) 103 Nm2 C–1 (2) 103 CN–1 m–2 (3) 6.32  103 Nm2 C–1 (4) 6.32  103 CN–1 m–2

E
16 Physics
8. An electric dipole is put in north-south direction in a sphere filled with water. Which statement is
correct?
(1) Electric flux is coming towards sphere
(2) Electric flux is coming out of sphere
(3) Electric flux entering into sphere and leaving the sphere are same
(4) Water does not permit electric flux to enter into sphere
9. Gauss's law is valid for :
(1) any closed surface (2) only regular closed surface
(3) any open surface (4) only irregular open surfaces
10. The electric field intensity at distance r and 2r from a long, uniformly charged straight wire at E1
and E2 respectively. The ratio of E1 and E2 will be :
1 2 1
(1) (2) (3) (4) None of these
2 1 1
11. When a body is charged, its mass :
(1) increases (2) decreases
(3) remains same (4) may increase or decrease

ASSERTION & REASON QUESTIONS (12 to 16)


In the following questions an Assertion (A) is followed by a corresponding Reason (R). Use the
following keys to choose the appropriate answer.
(1) Both (A) and (R) are true, (R) is the correct explanation of (A).
(2) Both (A) and (R) are true, (R) is not the correct explanation of (A).
(3) (A) is true; (R) is false.
(4) (A) is false; (R) is false.
12. Assertion (A) : Sharper is the curvature of spot on a charged body lesser will be the surface
charged density at that point.
Reason (R): Electric field is non-zero inside a charged conductor.
13. Assertion (A) : Charging is due to transfer of electrons.
Reason (R): Mass of a body decreases slightly when it is negatively charged.
14. Assertion (A) : Range of coulomb force is infinite.
Reason (R): Coulomb force acts between two charged particles.
15. Assertion (A) : If a point charge be rotated in a circle around a charge, the work done will be zero.
Reason (R): Work done is equal to dot product of force and distance.
16. Assertion (A) : A point charge is lying at the centre of a cube side. The electric flux emanating
th
1
from each surface of the cube is of total flux.
6
Reason (R): According to Gauss theorem, total electric flux through a closed surface enclosing a
1
charge is equal to times the magnitude of the charge enclosed.
0

ANSWER KEY
1. (4) 2. (1) 3. (2) 4. (1) 5. (3) 6. (1) 7. (1) 8. (3)
9. (1) 10. (2) 11. (4) 12. (4) 13. (3) 14. (2) 15. (1) 16. (2)

E
Physics 17

CHAPTER – 2
ELECTROSTATIC POTENTIAL & CAPACITANCE
1. ELECTROSTATIC POTENTIAL
It is the work done against the field to take a unit positive charge from infinity (reference point)
to the given point without gaining any kinetic energy.
 (W ) 
VP   P ext 
 q 
SI unit = Volt (V)
Potential at a point due to positive charge is positive & due to negative charge is negative.

Potential Due to Special Charge Distribution :

kq
(i) Point charge V
r

(ii) Charged conducting sphere


kq
(a) VC  ; r>R
r
kq
(b) VB  ; r=R
R
kq
(c) VA  ; r<R
R

2. POTENTIAL DIFFERENCE
The potential difference between two points A and B is work done by external agent against
electric field in taking a unit positive charge from B to A without acceleration (or keeping
(WBA )ext
Kinetic Energy constant) VA  VB  .
q
 Potential difference between two points in an electric field does not depend on the path between
them.

3. ELECTRIC POTENTIAL ENERGY OF TWO CHARGES
It is the amount of work required to bring the two point charges to a particular separation from
infinity without change in kinetic energy.
1 q1q2
U
40 r

E
18 Physics
4. EQUIPOTENTIAL SURFACE AND EQUIPOTENTIAL REGION
In an electric field the locus of points of equal potential is called an equipotential surface. An
equipotential surface and the electric field line meet at right angles. The region where E = 0,
Potential of the whole region must remain constant as no work is done in displacement of charge
in it, it is called as equipotential region like conducting bodies.

5. POTENTIAL GRADIENT
dV
| E |
dr
Electric potential goes on decreasing when we go in the direction of electrostatic field.

6. ELECTROSTATICS OF CONDUCTORS
• Electric field in the bulk of the conductor (volume) is zero while it is perpendicular to the surface
in electrostatics.
• Excess charge resides on the free surface of conductor in electrostatic condition.
• Potential throughout the volume of the conductor is same in electrostatics.

• Electric field at the surface of a charge conductor is given as E  nˆ
0
Electrostatics Shielding
• A metallic shield in form of a hollow conducting shell built to block an electric field because in a
hollow conducting shell, the electric field is zero at every point. This method of protection is
called electrostatics shielding.

7. DIELECTRICS AND POLARISATION


Dielectrics are non conducting substances. A dielectric (whether polar or non polar) develops a
net dipole moment in the presence of an external field. The dipole moment per unit volume is
called polarisation (P).
P  0e E
Dielectric strength of material : The minimum electric field required to ionize the medium or
the maximum electric field which the medium can bear without break down.
Dielectric strength of air = 3 × 106 V/m.

8. CAPACITOR & CAPACITANCE


A capacitor consists of two conductors carrying charges of equal magnitude and opposite sign.
The capacitance C of any capacitor is the ratio of magnitude of the charge Q on either conductor
Q
to the potential difference V between them  C 
V
unit of capacitance = farad (F).
If an isolated conductor is brought near to charged conductor then it increases the capacity of
system. This is the principle of capacitor.
The capacitance depends on the geometry of the conductors & presence of dielectric medium. It
doesn't depend on the charge of conductor or potential difference.

E
Physics 19
Capacitance of an isolated Spherical Conductor
C = 40r R in a medium C = 40 R in air

8.1 Parallel Plate Capacitor


(i) Uniform Di-electric Medium : If two parallel plates each of area A & separated by a
distance d are charged with equal & opposite charge Q, then the system is called a parallel
0 r A A
plate capacitor & its capacitance is given by ; C  in a medium; C  0 with air
d d
as medium.
This result is only valid when the electric field between plates of capacitor is constant.
0 A
(ii) Medium Partly Air : C 
 t 
d t  
 r 
When a di-electric slab of thickness t & relative permittivity
r is introduced between the plates of an air capacitor, then
the distance between the plates is effectively reduced by
 t 
 t   irrespective of the position of the di-electric slab.
 r 
On filling the space between the plates of a parallel plate
air capacitor with a dielectric, capacity of the capacitor is
increased because the same amount of charge can be stored
at a reduced potential.

8.2 Combination Of Capacitors


(i) Capacitors in Series : In this arrangement all the
capacitors when uncharged get the same charge Q but
the potential difference across each will differ if the
capacitance are unequal. In series combination the
potential difference on capacitor will be inversely
proportional to its capacitance.
1 1 1 1 1
    ..... 
Ceq. C1 C2 C3 Cn
Q = (Ceq.V) = C1V1 = C2V2 = C3V3

(ii) Capacitors In Parallel : When one plate of each


capacitor is connected to the positive terminal of the
battery & the other plate of each capacitor is connected to
the negative terminals of the battery, then the capacitors
are said to be in parallel connection. The capacitors

E
20 Physics
have the same potential difference, V but the charge on
each one is different (if the capacitors are unequal).
In parallel combination charge on capacitor will be distributed in the ratio of their
capacitance.
Ceq. = C1 + C2 + C3 + ...... + Cn .
Q = Q1 + Q2 + Q3
Q Q1 Q2 Q3
V   
Ceq C1 C2 C3

8.3 Energy Stored in a charged capacitor :


1 1
Capacitance C, charge Q & potential difference V; then energy stored is U = CV2 = QV =
2 2
1 Q2
where Q & V are charge & voltage of the capacitor C.
2 C
This energy is stored in the electrostatic field set up in the di-electric medium between the
conducting plates of the capacitor.
1
Energy density u  0 E2 , E is electric field between plate of capacitor.
2
Q2
Work done by battery to charge a capacitor W  CV2  QV 
C
8.4 Sharing Of Charges :
When two charged conductors of capacitance C1 & C2 at potential V1 & V2 respectively are
connected by a conducting wire, the charge flows from higher potential conductor to lower
potential conductor, until the potential of the two condensers becomes equal. The common
potential (V) after sharing of charges;
(i) When the positive plate of one connected to positive plate of other then common potential.
net ch arg e q  q2 C1V1  C2 V2
V  1 
net capaci tan ce C1  C2 C1  C2
Charges after sharing q1 = C1V & q2 = C2V. In this process energy is lost in the
connecting wire as heat.
C1 C2
This loss of energy is Uinitial  Ufinal  (V  V2 )2
2  C1  C2  1
C1V1  C2 V2
(ii) When positive of one with negative of other then common potential. VC  .
C1  C2
q1 C1
After connection ratio of charge would be in ratio of capacitance  .
q2 C2

E
Physics 21
PRACTICE QUESTIONS
CASE STUDY BASED QUESTIONS
Type -1

Equipotential Surface

v3
v2
v1 Equipotential surface

For the various charge systems, we represent equipotential surface by curves and line of force by
full line curves. Between any two adjacent equipotential surfaces. We assume a constant
potential difference the equipotential surface of a single point charge are concentric spherical
shells with their centres at the point charge. As the lines of force point radially outwards, so they
are perpendicular to the equipotential surface at all points.
1. Identify the wrong statement.
(1) Equipotential surface due to a single point charge is a straight line.
(2) Equipotential surface can be constructed for dipoles too.
(3) The electric field is normal to the equipotential surface through the point
(4) The work done to move a test charge on the equipotential surface is positive.
2. Nature of equipotential surface for a point charge is
(1) Ellipse with charge at foci
(2) Sphere with charge at the centre of the sphere
(3) Sphere with charge on the surface of the sphere
(4) Plane with charge on the surface.
3. A spherical equipotential surface is not possible
(1) Inside a uniformly charged sphere (2) For a dipole
(3) inside spherical condenser (4) for a point charge.
4. The work done in carrying a charge q once round a circle of radius a with a charge Q at its centre
is
qQ qQ q
(1) (2) (3) (4) zero
40a 40a 2 40a
5. For any charge configuration, equipotential surface through a point is ______ to the electrical
field at that point :
(1) perpendicular (2) at 45 degree (3) anti–parallel (4) parallel

E
22 Physics
Type 2 : Assertion and Reason :-
Question 6 to 11:-Note:- In the following questions an Assertion (A) is followed by a corresponding
Reason (R). Use the following keys to choose the appropriate answer.
(1) Both (A) and (R) are true, (R) is the correct explanation of (A).
(2) Both (A) and (R) are true, (R) is not the correct explanation of (A).
(3) (A) is true; (R) is false.
(4) (A) is false; (R) is false.
6. Assertion : If three capacitors of capacitance C1 < C2 < C3 are connected in parallel then their
equivalent capacitance Cp > Cs
1 1 1 1
Reason :   
Cp C1 C2 C3

7. Assertion : Electrons move away from a low potential to high potential region.
Reason : Because electrons has negative charge
8. Assertion : If the distance between parallel plates of a capacitor is halved and dielectric constant
is made three times, then the capacitor becomes 6 times.
Reason : Capacity of the capacitor does depend upon the nature of the material.
9. Assertion : A parallel plate capacitor is connected across battery through a key. A dielectric slab
of constant K is introduced between the plates. The energy which is stored becomes K times.
Reason : The surface density of charge on the plate remains constant or unchanged.
10. Assertion : When charges are shared between any two bodies, no charge is really lost, but some
loss of energy does occur.
Reason : Some energy disappears in the form of heat, sparking etc.
11. Assertion : Surface of a symmetrical conductor can be treated as equipotential surface.
Reason : Charges can easily flow in a conductor.

Type 3 : Multiple choice question (one correct answer) (Questions 12 to 21):-


12. Electric charges of 10C,  5C,  3C and 8C are placed at the corners of a square of side
2 m. the potential at the centre of the square is :
(1) 1.8 V (2) 1.8  106 V (3) 1.8  105 V (4) 1.8  104 V
13. An electric dipole of moment p is placed normal to the lines of force of electric intensity E ,
then the work done in deflecting it through an angle of 180 is :
(1) pE (2)  2pE (3) 2pE (4) Zero

E
Physics 23
14. Two opposite and equal charges 4  108 coulomb when placed 2  102 cm away, form a dipole.

If this dipole is placed in an external electric field 4  108 newton / coulomb , the value of
maximum torque and the work done in rotating it through 180 will be :

(1) 64  104 Nm and 64  104 J (2) 32  104 Nm and 32  104 J

(3) 64  104 Nm and 32  104 J (4) 32  104 Nm and 64  104 J


15. The capacitance of an air capacitor is 15F the separation between the parallel plates is 6mm . A

copper plate of 3mm thickness is introduced symmetrically between the plates. The capacitance
now becomes :
(1) 5F (2) 7.5F (3) 22.5F (4) 30F

16. A 2F capacitor is charged to 100 volt and then its plates are connected by a conducting wire.

The energy stored is :


(1) 1 J (2) 0.1 J (3) 0.01 J (4) 0.001 J

17. What is the value of capacitance of a capacitor which has a voltage of 4V and has 16 C of
charge?
(1) 2F (2) 4F (3) 6F (4) 8F
18. Capacitor is a device used to :
(1) Store electrical energy (2) Vary the resistance
(3) Store magnetic energy (4) dissipate energy
19. A 10 F, 20 F, 22F and 100 F capacitor are in parallel. The total capacitance is :
(1) 2.43 F (2) 4.86 F (3) 100 F (4) 152 F
20. When the plate area of a capacitor increases :
(1) the capacitor increases (2) the capacitance decreases
(3) the capacitance is unaffected (4) the voltage it can with stand
21. Assuming the capacitance of earth to be a spherical conductor of radius 6400 km is :
(1) 711 F (2) 644 F (3) 2000 F (4) 200 F

ANSWER KEY
1. (4) 2. (2) 3. (2) 4. (4) 5. (2) 6. (1) 7. (1) 8. (2)
9. (3) 10. (2) 11. (1) 12. (3) 13. (4) 14. (4) 15. (4) 16 (3)
17. (2) 18. (1) 19. (4) 20. (1) 21. (1)

E
24 Physics
ASSIGNMENT-1
(Electric Potential, Potential Due to Point charge and Electric Dipole)
1. Two charged spheres of radii 10 cm and 15 cm are connected by a thin wire. No current will
flow, if they have :
(1) The same charge on each (2) The same potential
(3) The same energy (4) The same field on their surfaces
2. Two electric charges 12C and 6C are placed 20 cm apart in air. There will be a point P on
the line joining these charges and outside the region between them, at which the electric potential
is zero. The distance of P from 6C charge is :
(1) 0.10 m (2) 0.15 m (3) 0.20 m (4) 0.25 m
3. Two small spheres each carrying a charge q are placed r metre apart. If one of the spheres is
taken around the other one in a circular path of radius r , the work done will be equal to :
(1) Force between them  r (2) Force between them  2r
(3) Force between them /2 r (4) Zero
4. A conductor with a positive charge :
(1) Is always at  ve potential (2) Is always at zero potential
(3) Is always at negative potential (4) May be at  ve , zero or ve potential
5. Equal charges are given to two spheres of different radii. The potential will :
(1) Be more on the smaller sphere
(2) Be more on the bigger sphere
(3) Be equal on both the spheres
(4) Depend on the nature of the materials of the spheres

ASSIGNMENT-2
(Equipotential Surface, Potential Gradient, Potential Energy)
6. Ten electrons are equally spaced and fixed around a circle of radius R. Relative to V = 0 at
infinity, the electrostatic potential V and the electric field E at the centre C are :
(1) V  0 and E  0 (2) V  0 and E  0 (3) V  0 and E  0 (4) V  0 and E  0
7. Angle between equipotential surface and lines of force is :
(1) Zero (2) 180 (3) 90 (4) 45
8. Electric potential at any point is V  5x  3y  15z , then the magnitude of the electric field is :
(1) 3 2 (2) 4 2 (3) 5 2 (4) 7
9. A uniform electric field having a magnitude E0 and direction along the positive X  axis exists.
If the potential V is zero at x  0 , then its value at X  x will be :
(1) V(x)   xE0 (2) Vx  xE0 (3) Vx  x2E0 (4) Vx  x2E0

E
Physics 25
10. Three charges Q,(q) and (q) are placed at the vertices of an equilateral triangle of side l as
shown in the figure. If the net electrostatic energy of the system is zero, then Q is equal to :
Q

l l

+q l +q

 q
(1)    (2) (q) (3) (q) (4) Zero
 2

ASSIGNMENT-3
(Electrostatics of Conductor, Capacitance, Parallel Plate Capacitor (Air))
11. The capacity of the conductor does not depend upon :
(1) Charge (2) Voltage
(3) Nature of the material (4) All of these
12. A conducting sphere of radius R  20 cm is given a charge Q  16C . What is E at centre :
(1) 3.6  106 N / C (2) 1.8  106 N / C (3) Zero (4) 0.9  106 N / C
13. In a charged capacitor, the energy resides :
(1) The positive charges (2) Both the positive and negative charges
(3) The field between the plates (4) Around the edge of the capacitor plates
14. The capacity of a parallel plate capacitor increases with the :
(1) Decrease of its area (2) Increase of its distance
(3) Increase of its area (4) None of the above
15. The capacity of a parallel plate condenser is 15 F , when the distance between its plates is 6 cm.
If the distance between the plates is reduced to 2 cm, then the capacity of this parallel plate
condenser will be :
(1) 15 F (2) 30 F (3) 45 F (4) 60 F

ASSIGNMENT-4
(Parallel Plate Capacitor (dielectric), Series and Parallel Combination of Capacitor,
Energy of capacitor, Energy Loss)
16. A capacitor is charged by using a battery which is then disconnected. A dielectric slab is then
slipped between the plates, which results in L :
(1) Reduction of charge on the plates and increase of potential difference across the plates
(2) Increase in the potential difference across the plate, reduction in stored energy, but no change
in the charge on the plates
(3) Decrease in the potential difference across the plates, reduction in the stored energy, but no
change in the charge on the plates
(4) None of the above

E
26 Physics
17. The capacity of a condenser is 4  106 farad and its potential is 100 volts . The energy released
on discharging it fully will be :
(1) 0.02 Joule (2) 0.04Joule (3) 0.025Joule (4) 0.05Joule
18. The capacity and the energy stored in a parallel plate condenser with air between its plates are
respectively Co and Wo . If the air is replaced by glass (dielectric constant = 5) between the
plates, the capacity of the plates and the energy stored in it will respectively be :
W0 Co Co Wo
(1) 5Co , 5Wo (2) 5Co , (3) , 5Wo (4) ,
5 5 5 5
19. Three capacitors are connected to D.C. source of 100 volts shown in the adjoining figure. If the
charge accumulated on plates of C1, C2 and C3 are qa , qb , qc ,qd .qe and q f respectively, then :
2F 3F 4F

a b c d e f

100 Volts

100
(1) q b  q d  q f  C (2) qb  qd  qf  0
9
(3) qa  qc  qe  50C (4) qb  qd  qf
20. Three capacitors each of capacitance 1F are connected in parallel. To this combination, a fourth
capacitor of capacitance 1F is connected in series. The resultant capacitance of the system is
4 3
(1) 4F (2) 2F (3) F (4) F
3 4

ANSWER KEY
1. (2) 2. (3) 3. (4) 4. (4) 5. (1) 6. (2) 7. (3) 8. (4)
9. (2) 10. (1) 11. (4) 12. (3) 13. (3) 14. (3) 15. (3) 16 (3)
17. (1) 18. (2) 19. (4) 20. (4)

E
Physics 27

EXERCISE-1

(Electric Potential, Potential Due to Point charge and Electric Dipole)


1. Inside a hollow charged spherical conductor, the potential :
(1) is constant
(2) varies directly as the distance from the centre
(3) varies inversely as the distance from the centre
(4) varies inversely as the square of the distance from the centre
2. Two charge  q and  q are situated at a certain distance. At the point exactly midway between
them :
(1) Electric field and potential both are zero
(2) Electric field is zero but potential is not zero
(3) Electric field is not zero but potential is zero
(4) Neither electric field nor potential is zero
3. The potential at a point, due to a positive charge of 100C at a distance of 9m, is :

(1) 104 V (2) 105 V (3) 106 V (4) 107 V


10
4. Charges of   109 C are placed at each of the four corners of a square of side 8cm . The
3
potential at the intersection of the diagonals is :
(1) 150 2 volt (2) 1500 2 volt (3) 900 2 volt (4) 900 volt
5. If a charged spherical conductor of radius 10cm has potential V at a point distant 5cm from its
centre, then the potential at a point distant 15cm from the centre will be :

1 2 3
(1) V (2) V (3) V (4) 3V
3 3 2

(Equipotential Surface, Potential Gradient, Potential Energy)


6. On rotating a point charge having a charge q around a charge Q in a circle of radius r. The work
done will be :
q  2Q Q
(1) q  2r (2) (3) Zero (4)
r 2 0 r

The electric potential V is given as a function of distance x (metre) by V  (5x  10x  9)volt .
2
7.
Value of electric field at x  1 is :
(1) 20V / m (2) 6V / m (3) 11V / m (4) 23V / m

E
28 Physics
8. Three particles, each having a charge of 10 C are placed at the corners of an equilateral triangle
of side 10cm . The electrostatic potential energy of the system is (Given
1
 9  109 N  m2 / C2 ) :
40
(1) Zero (2) Infinite (3) 27J (4) 100 J
9. Two positive point charges of 12C and 8C are 10cm apart. The work done in bringing them
4 cm closer is :
(1) 5.8 J (2) 5.8 eV (3) 13 J (4) 13 eV
10. Three charges Q,  q and  q are placed at the vertices of a right-angled isosceles triangle as
shown. The net electrostatic energy of the configuration is zero if Q is equal to :
Q

+q +q
a

q 2q
(1) (2) (3) 2q (4)  q
1 2 2 2

(Electrostatics of Conductor, Capacitance, Parallel Plate Capacitor (Air))


11. A cube of a metal is given a positive charge Q. For the above system, which of the following
statements is true :
(1) Electric potential at the surface of the cube is zero
(2) Electric potential within the cube is zero
(3) Electric field is normal to the surface of the cube
(4) Electric field varies within the cube
12. The capacity of a spherical conductor in SI system is :
R 4 0
(1) (2) (3) 40R (4) 40 R2
 0 R
13. A glass slab is put within the plates of a charged parallel plate which of the following quantities
does not change?
(1) Energy (2) Capacitance (3) Voltage (4) Charge
14. The radius of a metallic sphere if its capacitance is 1/ 9F , is :
6 9 8
(1) 10 m (2) 107 m (3) 10 m (4) 10 m
15. If the capacity of a spherical conductor is 1 pico-farad, then its diameter, would be :
(1) 1.8  103 m (2) 18  103 m (3) 1.8  105 m (4) 18  107 m

E
Physics 29
(Parallel Plate Capacitor (dielectric), Series and Parallel Combination of Capacitor,
Energy of Capacitor, Energy Loss)

16. The capacity of a parallel plate condenser is 5F . When a glass plate is placed between the
plates of the conductor, its potential becomes 1 / 8th of the original value. The value of dielectric
constant will be :
(1) 1.6 (2) 5 (3) 8 (4) 40
17. The insulated spheres of radii R1 and R 2 having charges Q1 and Q2 respectively are connected
to each other. There is :

(1) No change in the energy of the system


(2) An increase in the energy of the system
(3) Always a decrease in the energy of the system
(4) A decrease in the energy of the system unless Q1R2  Q2R1

18. In the circuit shown in the figure, the potential difference across the 4.5F capacitor is :
3F

4.5F

6F

12V

8
(1) volts (2) 4 volts (3) 6 volts (4) 8 volts
3
19. Four capacitors are connected as shown in the equivalent capacitance between the points P and Q
is:
1 3 4
(1) 4F (2) F (3) F (4) F
4 4 3
20. A 20F capacitor is charged to 5V and isolated. It is then connected in parallel with an uncharged
30F capacitor. The decrease in the energy of the system will be :
(1) 25 J (2) 200 J (3) 125 J (4) 150 J

ANSWER KEY
1. (1) 2. (3) 3. (2) 4. (2) 5. (2) 6. (3) 7. (1) 8. (3)
9. (3) 10. (2) 11. (3) 12. (3) 13. (4) 14. (3) 15. (2) 16 (3)
17. (4) 18. (4) 19. (4) 20. (4)

E
30 Physics
CASE STUDY BASED QUESTIONS
Energy stored in Capacitor
A capacitor is a device to store energy. The process of charging up a capacitor involves the
transferring of electric charges from its one place to another. This work done in charging the
capacitor is stored as its electrical potential energy.
Q –Q

1 2

If q is the charge and V is the potential difference across a capacitor at any instant during its
charging, then small work done in storing an additional small charge dq against the repulsion of
charge q already stored on it is dW = V.dq = (q/C)dq

1. A system of 2 capacitors of capacitance F and 4F is connected in series across a potential
difference of 6V. The energy stored in the system is ______.
(1) 3J (2) 24J (3) 30J (4) 108J
2. A capacitor of capacitance of 10F is charge to 10V. The energy stored in it is________.
(1) 100 J (2) 500 J (3) 1000 J (4) 1 J
3. A parallel plate air capacitor has capacity C farad, potential V volt and energy E Joule. When the
gap between the plates is completely filled with dielectric.
(1) both V and E increase (2) both V and E decrease
(3) V decreases, E increases (4) V increases, E decrease
4. A capacitor with capacitance 5F is charged to 5C. If the plates are pulled apart to reduce the
capacitance to 2F, how much work is done?
(1) 6.25 × 10–6 J (2) 3.75 × 10–6 J (3) 2.16 × 10–6 J (4) 2.55 × 10–6 J
5. A metallic sphere of radius 18 cm has been given a charge of 5 × 10–6C. the energy of the
charged conductor is ________
(1) 0.2 J (2) 0.6 J (3) 1.2 J (4) 2.4 J

ANSWER KEY
1. (2) 2. (2) 3. (2) 4. (4) 5. (2)

E
Physics 31
ASSERTION AND REASON QUESTIONS
Directions (Q. Nos. 1-8)
In the following questions an Assertion (A) is followed by a corresponding Reason (R). Use the
following keys to choose the appropriate answer.
(1) Both (A) and (R) are true, (R) is the correct explanation of (A).
(2) Both (A) and (R) are true, (R) is not the correct explanation of (A).
(3) (A) is true; (R) is false.
(4) (A) is false; (R) is false.
1. Assertion : The capacity of a given conductor remains same even if charge is varied on it.
Reason : Capacitance depends upon nearly medium as well as size and shape of conductor.
2. Assertion : If a point charge q is placed in front of an infinite grounded conducting plane
surface, the point charge will experience a force.
Reason : This force is due to the induced charge on the conducting surface potential.
3. Assertion : Dielectric polarisation means formation of positive and negative charge insides the
dielectric.
Reason : Free electrons are formed in this process.
4. Assertion : Two equipotential surface cannot cut each other.
Reason : Two equipotential surface are parallel to each other.
5. Assertion : A point charge is brought in an electric field. The field at a nearby point will
increase, whatever be the nature of the charge.
Reason : The electric field is independent of the nature of charge.
6. Assertion (A) : The whole charge of a conductor cannot be transferred to another isolated
conductor.
Reason (R): The total transfer of charge from one to another is not possible.
7. Assertion (A) : Electric field inside a conductor is zero.
Reason (R): The potential at all the points inside a conductor is same.
8. Assertion (A) : When air between the plates of a parallel plate condenser is replaced by an
insulating medium of dielectric constant its capacity increases.
Reason (R): Electric field intensity between the plates with dielectric in between it is reduced.

ANSWER KEY
1. (1) 2. (1) 3. (3) 4. (3) 5. (4) 6. (4) 7. (2) 8. (1)

E
32 Physics
EXERCISE-2
The electric potential V at any point O (x, y, z all in metres) in space is given by V  4x volt .
2
1.
The electric field at the point (1m,0,2m) in volt / metre is :

(1) 8 along negative X  axis (2) 8 along positive X  axis


(3) 16 along negative X  axis (4) 16 along positive Z  axis
2. Four equal charges Q are placed at the four corners of a square of each side is 'a ' . Work done in
removing a charge – Q from its centre to infinity is :

2Q 2 2Q 2 Q2
(1) 0 (2) (3) (4)
4 0a  0a 2 0a
3. In Millikan's oil drop experiment an oil drop carrying a charge Q is held stationary by a potential
difference 2400 V between the plates. To keep a drop of half the radius stationary the potential
difference had to be made 600 V . What is the charge on the second drop :

Q Q 3Q
(1) (2) (3) Q (4)
4 2 2
4. Two insulated charged conducting spheres of radii 20cm and 15cm respectively and having an
equal charge of 10C are connected by a copper wire and then they are separated. Then :

(1) Both the spheres will have the same charge of 10C

(2) Surface charge density on the 20cm sphere will be greater than that on the 15cm sphere

(3) Surface charge density on the 15cm sphere will be greater than that on the 20cm sphere

(4) Surface charge density on the two spheres will be equal


5. Two point charges 100  C and 5  C are placed at points A and B respectively with
AB  40cm . The work done by external force in displacing the charge 5  C from B to C ,
 1
where BC  30cm , angle ABC  and  9  109 Nm2 / C2 :
2 40

81 9 9
(1) 9J (2) J (3) J (4)  J
20 25 4
6. The electric potential at the surface of a charged solid sphere is 20 V. The value of electric
potential at its centre will be :
(1) 30 V (2) 20 V (3) 40 V (4) Zero

E
Physics 33
7. Capacitance of a parallel plate capacitor becomes 4/3 times its original value if a dielectric slab
of thickness t = d/2 is inserted between the plates (d is the separation between the plates). The
dielectric constant of the slab is :
(1) 8 (2) 4 (3) 6 (4) 2
8. A spherical drop of capacitance 1 F is broken into eight drops of equal radius. Then, the
capacitance of each small drop is __________.

1 1 1
(1) F (2) 8F (3) F (4) F
8 2 4

9. Two identical capacitors, have the same capacitance C. One of them is charged to potential V1 and

the other to V2 . The negative ends of the capacitors are connected together. When the positive ends

are also connected, the decrease in energy of the combined system is :


1 1 1 1
C  V1  V2  C  V1  V2 
2 2
(1) C(V12  V22 ) (2) C(V12  V22 ) (3) (4)
4 4 4 4
10. Three capacitors each of capacity 4 F are to be connected in such a way that the effective
capacitance is 6 F. This can be done by :
(1) connecting all of them in series (2) connecting them in parallel
(3) connecting two in series and one in parallel (4) connecting two in parallel and one in series
11. To form a composite 16F,1000V capacitor from a supply of identical capacitors marked
8F, 250V , we require a minimum number of capacitors :
(1) 40 (2) 2 (3) 8 (4) 2

ASSERTION & REASON QUESTIONS (12 to 16)


In the following questions an Assertion (A) is followed by a corresponding Reason (R). Use the
following keys to choose the appropriate answer.
(1) Both (A) and (R) are true, (R) is the correct explanation of (A).
(2) Both (A) and (R) are true, (R) is not the correct explanation of (A).
(3) (A) is true; (R) is false.
(4) (A) is false; (R) is false.
12. Assertion (A) : A capacitor is connected to a battery. If we move its plate further apart, work
will be done against the electrostatic attraction between the plates and the energy of the capacitor
gets decreased.
Reason (R): The energy stored in capacitor is dissipated in the form of heat energy.

E
34 Physics
13. Assertion (A) : Metallic shield in form of a hollow shell may be built to block on electric field.
Reason (R): In a hollow spherical shell. The electric field inside it is zero at every point.
14. Assertion (A) : If the distance between parallel pates of a capacitor is halved and dielectric
constant is made three times, then the capacitance becomes six time.
Reason (R): Capacitance of the capacitor does not depend upon the nature of the material of the
plates.
15. Assertion (A) : Charge on all the condensers connected in series is the same.
Reason (R): Capacitance of capacitor is directly proportional to charge on it.
16. Assertion (A) : Conductors having equal positive charge and volume, must also have same
potential.
Reason (R): Potential depends only on charge and volume of conductor.

ANSWER KEY
1. (1) 2. (3) 3. (2) 4. (3) 5. (4) 6. (2) 7. (4) 8. (3)
9. (3) 10. (3) 11. (2) 12. (2) 13. (1) 14. (3) 15. (3) 16. (4)

E
Physics 35

UNIT – II : CURRENT ELECTRICITY


CHAPTER – 3 : CURRENT ELECTRICITY

1. ELECTRIC CURRENT (I)


The rate of flow of electric charge across any cross-section is called electric current.
dq
(a) Instantaneous electric current I =
dt
Dq
(b) Average electric current Iav =
Dt

2. CURRENT DENSITY (J)


Current flowing per unit area through any cross-section is called current density.

I A I r r
J= I = J.A = JA cos q
A

3. DRIFT VELOCITY (Vd)


Average velocity with which electrons drift from low potential end to high potential end of the
r et r
conductor (vd). Drift velocity is given by v d = - E (in terms of applied electric field)
m
I
Vd = (in terms of current through the conductor)
neA
I = neAvd where A is the area of cross-section and “Avd” represents the rate of flow.

vd v et
Mobility : The term is called mobility of charge carriers, represented by m = d = .
E E m
SI unit of mobility = m2/Vs
node06\B0BA-BB\Kota\Board Material\Physics\CBSE\Booklet\Class XII\English\Term-I\3-Current Electricity

4. OHM’S LAW
V 1 l rl 1
I = where R = = where r (resistivity) =
R sA A s
Hence according to Ohm’s law when R is constant I µ V Þ I ~V curve is a straight line (at
constant temperature).
rl ml
• Resistance of a conductor is given by R = = 2
A ne tA
where r is resistivity. Its units is W-m.

E
36 Physics
m
• Resistivity of a conductor, r = (where m is mass of electron, n is number density of free
ne 2 t
electrons, t is average relaxation time).
l
Variation of resistance with length : R = r
A
(a) If a wire is cut to alter its length, then area remains same.
\ Rµl
(b) If a wire is stretched or drawn out or folded, area varies but volume remains constant.
\ R µ l2
Variation with area of cross-section or thickness
(a) If area is increased / decreased but length is kept same.
1 1
\ R µ or R µ 2 (r = radius / thickness)
A r
(b) If area is increased/decreased but volume remains same.
1 1
R µ 2 or R µ 4
A r

5. TEMPERATURE DEPENDENCE OF RESISTANCE AND RESISTIVITY


(1) For conductor : For conductor, resistance and
resistivity increases while increasing the temperature
rt = r0(1+ at), where ‘a’ is temperature coefficient
of resistivity.
As R µ r Þ R = R0(1 + a t)
(R0 is the resistance at reference temperature)
At temperature t1, R1 = R0 (1 + at1)
At temperature t2, R2 = R0 (1 + at2) Resistivity rT of copper as a

R 2 - R1 function of temperature T
Þ a=
R 0 (t 2 - t1 )
node06\B0BA-BB\Kota\Board Material\Physics\CBSE\Booklet\Class XII\English\Term-I\3-Current Electricity

(2) For Semiconductor


r

T
Temperature dependence of resistivity for a typical semiconductor.
For Semiconductors resistance and resistivity decreases while increasing the temperature.
(3) For Alloy
Alloys (like nicrome and manganin etc) exhibit a very weak dependency of resistivity with
temperature.

E
Physics 37
6. ELECTRIC ENERGY & ELECTRIC POWER
Electric Energy :
For a charge q, moving with potential V, the electric energy is expressed as qV or Vit.
When a current is passed through a resistor energy is wasted in over coming the resistances of the
wire. This energy is converted into heat (Heating effect of electric current).
V2
W = VIt = I2 Rt = t
R
SI unit = Joule (J)

Commercial unit of electrical energy consumption :


1 unit of electrical energy
= kilowatt hour
= 1 kWh = 3.6 × 106 joules.

Electric power :
The energy liberated per second in a device is called its power. The electrical power P delivered
by an electrical device is given by P = VI, where V = potential difference across device &
I = current. If the current enters the higher potential point of the device then power is consumed
by it (i.e. acts as load) . If the current enters the lower potential point then the device supplies
power (i.e. acts as source).
Power consumed by a resistor
V2
P = I2R = VI =
R
SI unit = Watt (W)

7. CELL
• EMF (E) : The potential difference across the terminals of a practical cell when no current is
being drawn from it.
• Internal Resistance (r) : The opposition of flow of current inside the cell. It depends on
(i) Distance between electrodes ­ Þ r­
node06\B0BA-BB\Kota\Board Material\Physics\CBSE\Booklet\Class XII\English\Term-I\3-Current Electricity

(ii) Area of electrodes ­ Þ r¯


(iii) Concentration of electrolyte ­ Þ r­
(iv) Temperature ­ Þ r¯

Series grouping
E1 E2 E3
+ – r1 + – r2 + – r3 n cells

E
38 Physics
(a) Eequivalent = E1 + E2 + E3 + ....... En

(b) requivalent = r1 + r2 + r3 + ...... rn

(c) Current i = åE i

år +R
i

nE
(d) If all cells have equal emf E and equal internal resistance r then i =
nr + R
Cases :
E nE
(i) If nr >> R Þ i = (ii) If nr << R Þ i =
r R

Parallel Grouping
E1 E 2 E 3 E1 r1
+ + + ......
r1 r2 r3 E2
(a) Eequivalent = r2
1 1 1
+ + + ........ E3
r1 r2 r3 r3

1
(b) requivalent =
1 1 1 n cell
+ + + ...... i
r1 r2 r3

R
(c) If all cells have equal emf. E and internal resistance r then Eequivalent = E
r E
requivalent = Þ current i = r
n n +R

Mixed combination
Total number of identical cell in this circuit is nm. If n cells connected in a series and there are m
such branches in the circuit than the internal resistance of the cells connected in a row = nr
E r E r E r

E r E r E r
node06\B0BA-BB\Kota\Board Material\Physics\CBSE\Booklet\Class XII\English\Term-I\3-Current Electricity

E r E r E r

R
1 1 1
Total internal resistance of the circuit = + + ....upto m times
rnet nr nr
nr
(Q There are such m rows) rnet =
m

E
Physics 39
Total e.m.f. of the circuit = total e.m.f. of the cells connected in a row ET = nE
E net nE
I= =
R + rnet R + nr
m
Condition for maximum power transfer :
req = R e q

8. KIRCHHOFF’S LAWS
(i) Junction Rule :
It is based on conservation of charge.

(ii) Loop Rule :


For any closed loop, total rise in potential + total fall in potential = 0.

R
+V – iR = 0
i
V
It is based on conservation of energy.

9. WHEAT STONE BRIDGE


P R
When current through the galvanometer is zero (null point or balance point) then = .
Q S
When PS > QR, VC < VD & PS < QR, VC > VD or PS = QR Þ products of opposite arms are
node06\B0BA-BB\Kota\Board Material\Physics\CBSE\Booklet\Class XII\English\Term-I\3-Current Electricity

equal. Potential difference between C & D at null point is zero . The null point is not affected by
resistance of G & E. It is not affected even if the positions of G & E are interchanged.
C
P Q

A B
G

R S
D
E R

E
40 Physics
10. METRE BRIDGE
Principle : It's principle is based on the balanced condition of wheat stone bridge.
At balance condition :
P R
= Þ
l R
= ÞS=
(100 - l ) R
Q S (100 - l) S l
S
RB
D
G
P J Q
B
A l (100-l) C

11. POTENTIOMETER
Principle : The potential difference between any segment of conductor is directly proportional to
the length of the segment. i.e. V µ L
A potentiometer is a linear conductor of uniform cross-section with a steady current set up in it.
This maintains a uniform potential gradient along the length of the wire . Any potential
difference which is less than the potential difference maintained across the potentiometer wire
can be measured using this.
Circuits of potentiometer :
E r Rh(0 –R)

primary circuit
L
A B
secondary circuit wire

E' G E'<E

current ´ resistance of
V potentiometer wire æRö
K(potential gradient) = = = Iç ÷
L length of potentiometer wire èLø
node06\B0BA-BB\Kota\Board Material\Physics\CBSE\Booklet\Class XII\English\Term-I\3-Current Electricity

Applications of potentiometer :
E1 l1
(i) Comparison of emfs of two cells =
E2 l 2

J J'
A B
E1

1
2
3 G
E2

E
Physics 41
(ii) Internal Resistance of a given primary cell
æl -l ö
r = ç 1 2 ÷R
è l2 ø
Rh

J' J
A B
E

R G
R.B.
K

12. IMPORTANT POINTS


(i) A current flows through a conductor only when there is an electric field within the conductor
because the drift velocity of electrons is directly proportional to the applied electric field.
(ii) A metal has a resistance and gets often heated by flow of current because when free electrons
drift through a metal, they make occasional collisions with the lattice. These collisions are
inelastic and transfer energy to the lattice as internal energy.
(iii) Potentiometer is an ideal instrument to measure the potential difference because it does not draws
any current from source.
(iv) An ammeter is always connected in series whereas a voltmeter is connected in parallel
because an ammeter is a low-resistance galvanometer while a voltmeter is a high-resistance
galvanometer.
node06\B0BA-BB\Kota\Board Material\Physics\CBSE\Booklet\Class XII\English\Term-I\3-Current Electricity

E
42 Physics

PRACTICE QUESTIONS
CASE STUDY BASED QUESTIONS
Type -1

(Potentiometer: An Ideal Voltmeter)


Potentiometer is an apparatus used for measuring the emf of a cell or potential difference between
two points in an electrical circuit accurately. It is also used to determine the internal resistance of
a primary cell. The potentiometer is based on the principle that, if V is the potential difference
across any portion of the wire of length l and resistance R, then V µ l or V = k l where k is the
potential gradient. Thus, potential difference across any portion of potentiometer wire is directly
proportional to length of the wire of that portion. The potentiometer wire must be uniform. The
resistance of potentiometer wire should be high.
e r Rh=(0- R1)
( )
Primary circuit
L
A B
Secondary circuit Wire
e'< e

e' G
1. Which one of the following is true about potentiometer?
(1) Its sensitivity is low.
(2) It measures the emf of a cell very accurately.
(3) It is based on deflection method.
(4) None of the above
2. A current of 1.0 mA is flowing through a potentiometer wire of length 4 cm and of resistance
4 W. The potential gradient of the potentiometer wire is
(1) 10–3 V m–1 (2) 10–5 V m–2 (3) 2 × 10–3 V m–1 (4) 4 × 10–3 V m–1
3. Sensitivity of a potentiometer an be increased by
(1) decreasing potential gradient along the wire
(2) increasing potential gradient along the wire
(3) decreasing current through the wire
(4) increasing current through the wire
node06\B0BA-BB\Kota\Board Material\Physics\CBSE\Booklet\Class XII\English\Term-I\3-Current Electricity

4. A potentiometer is an accurate and versatile device to make electrical measurements of EMF


because the method involves
(1) potential gradients
(2) a condition of no current flow through the galvanometer
(3) a combination of cells, galvanometer and resistances
(4) cells
5. In a potentiometer experiment, the balancing length is 8 m, when the two cells E1 and E2 are
joined in series. When the two cells are connected in opposition the balancing length is 4 m. The
ratio of the e.m.f. of two cells (E1/E2) is
(A) 1 : 2 (B) 2 : 1 (C) 1 : 3 (D) 3 : 1

E
Physics 43
Type 2 : Assertion and Reason :-
Question 6 to 10:-Note:- In the following questions an Assertion (A) is followed by a corresponding
Reason (R). Use the following keys to choose the appropriate answer.
(1) Both (A) and (R) are true, (R) is the correct explanation of (A).
(2) Both (A) and (R) are true, (R) is not the correct explanation of (A).
(3) (A) is true; (R) is false.
(4) (A) is false; (R) is false.
6. Assertion : The resistivity of a semiconductor increases with temperature.
Reason : The atoms of a semiconductor vibrate with larger amplitude at higher temperatures
thereby increasing its resistivity
7. Assertion : In a simple battery circuit the point of lowest potential is positive terminal of the
battery.
Reason : The current flows towards the point of the higher potential as it flows in such a circuit
from the negative to the positive terminal.
8. Assertion : The temperature coefficient of resistance is positive for metals and negative for
p-type semiconductor.
Reason : The effective charge carriers in metals are negatively charged whereas in p-type
semiconductor they are positively charged.
9. Assertion : Total current entering a circuit is equal to that leaving it by Kirchhoff's Law.
Reason : It is based on conservation of energy.
10. Assertion : There is no current in the metals in the absence of electric field.
Reason : Motion of free electron are randomly.
Type 3 : Multiple choice question (one correct answer) (Questions 11 to 21):-
11. A wire 100 cm long and 2.0 mm diameter has a resistance of 0.7 ohm, the electrical resistivity of
the material is
(1) 4.4 ´ 10 -6 ohm ´ m (2) 2.2 ´ 10-6 ohm ´ m (3) 1.1 ´ 10 -6 ohm ´ m (4) 0.22 ´ 10 -6 ohm ´ m
12. A wire of length 5 m and radius 1 mm has a resistance of 1 ohm. What length of the wire of the
same material at the same temperature and of radius 2 mm will also have a resistance of 1 ohm
(1) 1.25 m (2) 2.5 m (3) 10 m (4) 20 m
13. In the series combination of two or more than two resistances :
(1) the current through each resistance is same
(2) the voltage through each resistance is same
node06\B0BA-BB\Kota\Board Material\Physics\CBSE\Booklet\Class XII\English\Term-I\3-Current Electricity

(3) neither current nor voltage through each resistance is same


(4) both current and voltage through each resistance is same
14. Consider the circuit shown in the figure. The current I3 is equal to :
28W 54W

I1 6V I2
I3

8V 12 V

(1) 5 amp (2) 3 amp (3) -3amp (4) -5 / 6amp

E
44 Physics
15. Two sources of equal emf are connected to an external resistance R. The internal resistances of
the two sources are R1 and R 2 (R 2 > R1 ) . If the potential difference across the source having
internal resistance R 2 is zero, then
(1) R = R1R 2 / (R1 + R 2 ) (2) R = R1R 2 / (R 2 - R1 )
(3) R = R 2 ´ (R1 + R 2 ) / (R 2 - R1 ) (4) R = R 2 - R1
16. In Wheatstone's bridge P = 9 ohm, Q = 11 ohm, R = 4 ohm and S = 6 ohm. How much resistance
must be put in parallel to the resistance S to balance the bridge
44
(1) 24 ohm (2) ohm (3) 26.4 ohm (4) 18.7 ohm
9
17. Which of the following statement is wrong :
(1) Voltmeter should have high resistance
(2) Ammeter should have low resistance
(3) Ammeter is placed in parallel across the conductor in a circuit
(4) Voltmeter is placed in parallel across the conductor in a circuit
18. The resistance of an ideal voltmeter is :
(1) Zero (2) Very low (3) Very large (4) Infinite
19. The resistance of 10 metre long potentiometer wire is 1 ohm/meter. A cell of e.m.f. 2.2 volts and
a high resistance box are connected in series to this wire. The value of resistance taken from
resistance box for getting potential gradient of 2.2 millivolt/metre will be
(1) 790 W (2) 810 W (3) 990 W (4) 1000 W
20. With the three identical bulbs of 60 W, 200 V rating are connected in series to a 200 V, source
the power draw by them will be :
(1) 20 W (2) 60 W (3) 180 W (4) 10 W
21. The magnitude and direction of the current in the circuit shown will be :
1W e 2W
a b
10V 4V

3W
d c
node06\B0BA-BB\Kota\Board Material\Physics\CBSE\Booklet\Class XII\English\Term-I\3-Current Electricity

7 7
(1) A from a to b through e (2) A from b to a through e
3 3
(3) 1A from b to a through e (4) 1A from a to b through e

ANSWER KEY
1. (2) 2. (1) 3. (1) 4. (2) 5. (4) 6. (4) 7. (4) 8. (2)
9. (1) 10. (1) 11. (2) 12. (4) 13. (1) 14. (4) 15. (4) 16 (3)
17. (3) 18. (4) 19. (3) 20. (1) 21. (4)

E
Physics 45
ASSIGNMENT-1
(Current, Current Density, Ohm’s Law, Drift Velocity)

1. The specific resistance of manganin is 50 ´ 10 -8 ohm ´ m . The resistance of a cube (of


manganin) of length 50 cm will be :

(1) 10 -6 ohm (2) 2.5 ´ 10 -5 ohm (3) 10 -8 ohm (4) 5 ´ 10 -4 ohm

2. The specific resistance of a wire is r , its volume is 3 m3 and its resistance is 3 ohms, then its
length will be :
1 3 1 1
(1) (2) (3) 3 (4) r
r r r 3

3. The voltage V and current I graph for a conductor at two different temperatures T1 and T2 are
shown in the figure. The relation between T1 and T2 is :
T1
V
T2

(1) T1 > T2 (2) T1 » T2 (3) T1 = T2 (4) T1 < T2


4. When the length and area of cross-section both are doubled, then its resistance
(1) Will become half (2) Will be doubled
(3) Will remain the same (4) Will become four times
5. A current is passing through a wire having two sections P and Q of uniform diameters d and d/2
respectively. If the mean drift velocity of electrons in sections P and Q is denoted by vP and vQ
respectively, then
1 1
(1) vP = vQ (2) vP = vQ (3) vP = vQ (4) vP = 2 vQ
2 4
node06\B0BA-BB\Kota\Board Material\Physics\CBSE\Booklet\Class XII\English\Term-I\3-Current Electricity

ASSIGNMENT-2
(Temperature dependence of Resistance & Resistivity, Electrical Energy and Power)

6. At what temperature will the resistance of a copper wire become three times its value at 0oC
(Temperature coefficient of resistance for copper = 4 × 10–3 per oC )
(1) 400oC (2) 450oC (3) 500oC (4) 550oC
7. The specific resistance of all metals is most affected by
(1) Temperature (2) Pressure
(3) Degree of illumination (4) Applied magnetic field

E
46 Physics
8. If the resistance of a conductor is 5 W at 50oC and 7 W at 100oC then the mean temperature
coefficient of resistance of the material is
(1) 0.008/oC (2) 0.006/oC (3) 0.004/oC (4) 0.001/oC
9. A platinum resistance thermometer has a resistance of 50 W at 20°C . When dipped in a liquid
the resistance becomes 76.8 W . The temperature coefficient of resistance for platinum is
a = 3.92 ´ 10 -3 / °C . The temperature of the liquid is
(1) 100°C (2) 137°C (3) 167°C (4) 200°C
10. For driving a current of 2A for 6 minutes in a circuit, 1000J of work is to be done. The e.m.f. of
the source in the circuit is
(1) 1.38 V (2) 1.68 V (3) 2.04 V (4) 3.10 V
ASSIGNMENT-3
(Cell and Combinations of Cells)

11. In the shown circuit, what is the potential difference across A and B :
20 V

A B

(1) 50 V (2) 45 V (3) 30 V (4) 20 V


12. When cells are connected in parallel, then
(1) The current decreases (2) The current increases
(3) The e.m.f. increases (4) The e.m.f. decreases
13. n identical cells each of e.m.f. E and internal resistance r are connected in series. An external
resistance R is connected in series to this combination. The current through R is
nE nE E nE
(1) (2) (3) (4)
node06\B0BA-BB\Kota\Board Material\Physics\CBSE\Booklet\Class XII\English\Term-I\3-Current Electricity

R + nr nR + r R + nr R+r
14. A torch battery consisting of two cells of 1.45 volts and an internal resistance 0.15W , each cell
sending currents through the filament of the lamps having resistance 1.5 ohms. The value of
current will be
(1) 16.11 amp (2) 1.611 amp (3) 0.1611 amp (4) 2.6 amp
15. The electromotive force of a primary cell is 2 volts. When it is short-circuited it gives a current of
4 amperes. Its internal resistance in ohms is
(1) 0.5 (2) 5.0 (3) 2.0 (4) 8.0

E
Physics 47
ASSIGNMENT-4
(Kirchhoff’s law, Wheatstone bridge)
16. In the given current distribution what is the value of I :

4A
I 2A

3A
5A

(1) 3A (2) 8 A (3) 2A (4) 5A


17. The terminal potential difference of a cell is greater than its e.m.f. when it is
(1) Being discharged (2) In open circuit
(3) Being charged (4) Being either charged or discharged
18. Which is a wrong statement
(1) The Wheatstone bridge is most sensitive when all the four resistances are of the same order
(2) In a balanced Wheatstone bridge, interchanging the positions of galvanometer and cell affects
the balance of the bridge
(3) Kirchhoff's first law (for currents meeting at a junction in an electric circuit) expresses the
conservation of charge
(4) The rheostat can be used as a potential divider
19. Kirchhoff's first law i.e. Si = 0 at a junction is based on the law of conservation of
(1) Charge (2) Energy
(3) Momentum (4) Angular momentum
20. Which instrument is used as the null detector in wheatstone bridge?
(1) Voltmeter (2) Ammeter (3) Galvanometer (4) Multimeter

ASSIGNMENT-5
(Meter Bridge, Potentiometer)
node06\B0BA-BB\Kota\Board Material\Physics\CBSE\Booklet\Class XII\English\Term-I\3-Current Electricity

21. Constantan wire is used in making standard resistances because its


(1) Specific resistance is low
(2) Density is high
(3) Temperature coefficient of resistance is negligible
(4) Melting point is high
22. If the resistivity of a potentiometer wire be r and area of cross-section be A, then what will be
potential gradient along the wire
Ir I IA
(1) (2) (3) (4) IAr
A Ar r

E
48 Physics
23. When connected across the terminals of a cell, a voltmeter measures 5V and a connected
ammeter measures 10 A of current. A resistance of 2 ohms is connected across the terminals of
the cell. The current flowing through this resistance will be
(1) 2.5 A (2) 2.0 A (3) 5.0 A (4) 7.5 A
24. Sensitivity of potentiometer can be increased by
(1) Increasing the e.m.f. of the cell
(2) Increasing the length of the potentiometer wire
(3) Decreasing the length of the potentiometer wire
(4) None of the above
25. For measurement of potential difference, potentiometer is preferred in comparison to voltmeter
because
(1) Potentiometer is more sensitive than voltmeter
(2) The resistance of potentiometer is less than voltmeter
(3) Potentiometer is cheaper than voltmeter
(4) Potentiometer does not take current from the circuit

ANSWER KEY
1. (1) 2. (2) 3. (1) 4. (3) 5. (3) 6. (3) 7. (1) 8. (1)
9. (3) 10. (1) 11. (4) 12. (2) 13. (1) 14. (2) 15. (1) 16 (3)
17. (3) 18. (2) 19. (1) 20. (3) 21. (3) 22. (1) 23. (2) 24. (2)
25. (4)

node06\B0BA-BB\Kota\Board Material\Physics\CBSE\Booklet\Class XII\English\Term-I\3-Current Electricity

E
Physics 49

EXERCISE-1

(Current, Current Density, Ohm’s Law, Drift Velocity)


1. When the current i is flowing through a conductor, the drift velocity is v . If 2i current is flowed
through the same metal but having double the area of cross-section, then the drift velocity will be
(1) v / 4 (2) v / 2 (3) v (4) 4v
2. When current flows through a conductor, then the order of drift velocity of electrons will be
(1) 1010 m / sec (2) 10 -2 cm / sec (3) 10 4 cm / sec (4) 10 -1 cm / sec
3. Which of the adjoining graphs represents ohmic resistance :

V V V V

(1) (2) (3) (4)


I I I I

4. The resistivity of a wire


(1) Increases with the length of the wire
(2) Decreases with the area of cross-section
(3) Decreases with the length and increases with the cross-section of wire
(4) None of the above statement is correct
5. 5 amperes of current is passed through a metallic conductor. The charge flowing in one minute in
coulombs will be
(1) 5 (2) 12 (3) 1/12 (4) 300

(Temperature dependence of Resistance, Electrical Energy and Power)


6. The resistance of a coil is 4.2 W at 100o C and the temperature coefficient of resistance of its
material is 0.004/o C. Its resistance at 0o C is
(1) 6.5 W (2) 5 W (3) 3 W (4) 4 W
7. When a current flows through a conductor its temperature
node06\B0BA-BB\Kota\Board Material\Physics\CBSE\Booklet\Class XII\English\Term-I\3-Current Electricity

(1) May increase or decrease (2) Remains same


(3) Decreases (4) Increases
8. The temperature coefficient of resistance for a wire is 0.00125 / °C . At 300K its resistance is
1 ohm. The temperature at which the resistance becomes 2 ohm is
(1) 1154 K (2) 1100 K (3) 1400 K (4) 1127 K
9. The S.I. unit of power is
(1) Henry (2) Coulomb (3) Watt (4) Watt-hour
10. What is the electric current flowing in 40W and 220V bulb?
(1) 1.8 ampere (2) 18 ampere (3) 0.18 ampere (4) 0.018 ampere

E
50 Physics
(Cell and Combinations of Cells)
11. The internal resistance of a cell depends on
(1) The distance between the plates (2) The area of the plates immersed
(3) The concentration of the electrolyte (4) All the above
12. A current of two amperes is flowing through a cell of e.m.f. 5 volts and internal resistance
0.5 ohm from negative to positive electrode. If the potential of negative electrode is 10V, the
potential of positive electrode will be
(1) 5 V (2) 14 V (3) 15 V (4) 16 V
13. If six identical cells each having an e.m.f. of 6V are connected in parallel, the e.m.f. of the
combination is
1
(1) 1 V (2) 36 V (3) V (4) 6 V
6
14. A storage battery has e.m.f. 15 volts and internal resistance 0.05 ohm. Its terminal voltage
when it is delivering 10 ampere is
(1) 30 volts (2) 1.00 volts (3) 14.5 volts (4) 15.5 volts
15. If two identical cells each having an emf of 2V are connected in series. The emf of the
combination :
(1) 1 V (2) 4V (3) 8V (4) 6V

(Kirchhoff’s Law, Wheatstone Bridge)


16. The current in the arm CD of the circuit will be
B

i1 i2
O A
i3
C
D

(1) i1 + i 2 (2) i2 + i3 (3) i1 + i3 (4) i1 - i2 + i3


17. The figure shows a network of currents. The magnitude of currents is shown here. The current I
will be :
1A
node06\B0BA-BB\Kota\Board Material\Physics\CBSE\Booklet\Class XII\English\Term-I\3-Current Electricity

10 A I

6A

2A

(1) 3 A (2) 9 A (3) 13 A (4) 19 A


18. Kirchhoff's second law is based on the law of conservation of
(1) Charge (2) Energy
(3) Momentum (4) Sum of mass and energy

E
Physics 51
19. If in the experiment of Wheatstone's bridge, the positions of cells and galvanometer are
interchanged, then balance points will
(1) Change
(2) Remain unchanged
(3) Depend on the internal resistance of cell and resistance of galvanometer
(4) None of these
20. In the Wheatstone's bridge (shown in figure) X = Y and A > B . The direction of the current
between ab will be:
a
A B
c d

X Y
b

(1) From a to b (2) From b to a


(3) From b to a through c (4) From a to b through c
(Meter Bridge, Potentiometer)
21. The material of wire of potentiometer is
(1) Copper (2) Steel (3) Manganin (4) Aluminium
22. In a meter bridge, the balancing length from the left end (standard resistance of one ohm is in the
right gap) is found to be 20 cm. The value of the unknown resistance is
(1) 0.8W (2) 0.5W (3) 0.4 W (4) 0.25W
23. A potentiometer is used for the comparison of e.m.f. of two cells E1 and E 2 . For cell E1 the no
deflection point is obtained at 20 cm and for E 2 the no deflection point is obtained at 30 cm . The
ratio of their e.m.f.'s will be
(1) 2/3 (2) 1/2 (3) 1 (4) 2
24. Potential gradient is defined as
(1) Fall of potential per unit length of the wire
(2) Fall of potential per unit area of the wire
(3) Fall of potential between two ends of the wire
node06\B0BA-BB\Kota\Board Material\Physics\CBSE\Booklet\Class XII\English\Term-I\3-Current Electricity

(4) Potential at any one end of the wire


25. A Daniel cell is balanced on 125 cm length of a potentiometer wire. Now the cell is short-
circuited by a resistance 2 ohm and the balance is obtained at 100 cm . The internal resistance of
the Daniel cell is
(1) 0.5 ohm (2) 1.5 ohm (3) 1.25 ohm (4) 4/5 ohm
ANSWER KEY
1. (3) 2. (2) 3. (1) 4. (4) 5. (4) 6. (3) 7. (4) 8. (4)
9. (3) 10. (3) 11. (4) 12. (2) 13. (4) 14. (3) 15. (2) 16 (2)
17. (3) 18. (2) 19. (2) 20. (2) 21. (3) 22. (4) 23. (1) 24. (1)
25. (1)

E
52 Physics

CASE STUDY BASED QUESTIONS

(Mechanism of Current Flow in a Conductor)


Metals have a large number of free electrons nearly 1028 per cubic metre. In the absence of
electric field, average terminal speed of the electrons in random motion at room temperature is of
the order of 105 ms–1. When a potential difference V is applied across the two ends of a given
conductor, the free electrons in the conductor experiences a force and are accelerated towards the
positive end of the conductor. One their way, they suffer frequent collisions with the ions/atoms
of the conductor and lose their gained kinetic energy. After each collision, the free electrons are
again accelerated due to electric field, towards the positive end of the conductor and lose their
gained kinetic energy in the next collision with the ions/atoms of the conductor. The average
speed of the free electrons with which they drift towards the positive end of the conductor under
the effect of applied electric field called drift speed of the electrons.

e
I I
– +

1. Magnitude of drift velocity per unit electric field is :


(1) current density (2) current (3) resistivity (4) mobility
2. The drift speed of the electrons depends on :
(1) dimensions of the conductor
(2) number density of free electrons in the conductor
(3) both (1) and (2)
(4) neither (1) nor (2)
node06\B0BA-BB\Kota\Board Material\Physics\CBSE\Booklet\Class XII\English\Term-I\3-Current Electricity

3. We are able to obtain fairly large currents in a conductor because :


(1) the electron drift speed is usually very large
(2) the number density of free electrons is very high and this can compensate for the low values
of the electrons drift speed and the very small magnitude of the electron charge
(3) the number density of free electrons as well as the electron drift speeds are very large and
these compensate for the very small magnitude of the electron charge
(4) the very small magnitude of the electron charge has to be divided by the still smaller product
of the number density and drift speed to get the electric current

E
Physics 53
4. Drift speed of electrons in a conductor is very small i.e., i = 10–4 ms–1. The Electric bulb glows
immediately. When the switch is closed because :
(1) drift velocity of electron increases when switch is closed
(2) electrons are accelerated towards the negative end of the conductor
(3) the drifting of electrons takes place at the entire length of the conductor
(4) the electrons of conductor move towards the positive end and protons of conductor move
towards negative end of the conductor
5. The number density of free electrons in a copper conductor is 8.5 × 1028 m–3. How long does an
electron take to drift from one end of a wire 3.0 m long to its other end? The area of cross-section
of the wire is 2.0 × 10–6 m2 and its is carrying a current of 3.0 A :
(1) 8.1 × 104 s (2) 2.7 × 104 s (3) 9 × 103 s (4) 3 × 103 s

ANSWER KEY
1. (4) 2. (4) 3. (2) 4. (3) 5. (2)
node06\B0BA-BB\Kota\Board Material\Physics\CBSE\Booklet\Class XII\English\Term-I\3-Current Electricity

E
54 Physics
ASSERTION AND REASON QUESTIONS

Directions (Q. Nos. 1-8)


In the following questions an Assertion (A) is followed by a corresponding Reason (R). Use the
following keys to choose the appropriate answer.
(1) Both (A) and (R) are true, (R) is the correct explanation of (A).
(2) Both (A) and (R) are true, (R) is not the correct explanation of (A).
(3) (A) is true; (R) is false.
(4) (A) is false; (R) is false.
1. Assertion : Insulator do not allow flow of current through them.
Reason : Insulator have no free charge carrier.
2. Assertion : It is advantageous to transmit electric power at high voltage.
Reason : High voltage implies high current.
3. Assertion : A larger dry cell has higher emf.
Reason : The emf of a dry cell is proportional to its size.
4. Assertion : The current in a wire is due to flow of free electrons in a definite direction.
Reason : A current carrying wire should have non-zero charge.
5. Assertion : The conducting wires are made of copper.
Reason : The electrical conductivity of copper is high.
6. Assertion : Voltmeter is connected in parallel with the circuit.
Reason : Resistance of a voltmeter is very large.
7. Assertion : The drift velocity of electrons in a metallic wire will decrease, if the temperature of
the wire is increased.
Reason : On increasing temperature, conductivity of metallic wire decreases.
8. Assertion : The electric bulbs glows immediately when switch is on.
Reason : The drift velocity of electrons in a metallic wire is very high.
node06\B0BA-BB\Kota\Board Material\Physics\CBSE\Booklet\Class XII\English\Term-I\3-Current Electricity

ANSWER KEY
1. (1) 2. (3) 3. (4) 4. (3) 5. (1) 6. (2) 7. (2) 8. (3)

E
Physics 55

EXERCISE-2
1. Every atom makes one free electron in copper. If 1.1 ampere current is flowing in the wire of
copper having 1 mm diameter, then the drift velocity (approx.) will be (Density of copper
= 9 ´ 103 kg m -3 and atomic weight = 63)
(1) 0.3mm / sec (2) 0.1mm / sec (3) 0.2 mm / sec (4) 0.2 cm / sec
2. In an electrolyte 3.2 ´ 1018 bivalent positive ions drift to the right per second while 3.6 ´ 1018
monovalent negative ions drift to the left per second. Then the current is
(1) 1.6 amp to the left (2) 1.6 amp to the right
(3) 0.45 amp to the right (4) 0.45 amp to the left
3. Two wires A and B of same material and same mass have radius 2r and r. If resistance of wire A
is 34 W , then resistance of B will be
(1) 544 W (2) 272 W (3) 68 W (4) 17W
4. A source of e.m.f. E = 15 V and having negligible internal resistance is connected to a variable
resistance so that the current in the circuit increases with time as i = 1.2 t + 3. Then, the total
charge that will flow in first five second will be
(1) 10 C (2) 20 C (3) 30 C (4) 40 C
5. When a resistance of 2 ohm is connected across the terminals of a cell, the current is 0.5 A.
When the resistance is increased to 5 ohm, the current is 0.25 A. The e.m.f. of the cell is
(1) 1.0 V (2) 1.5 V (3) 2.0 V (4) 2.5 V
6. In the given circuit the current I1 is :
30 W

I1
40 W

I3
I2 40V
40 W

80V

(1) 0.4 A (2) – 0.4 A (3) 0.8 A (4) – 0.8 A


7. A cell of constant e.m.f. first connected to a resistance R1 and then connected to a resistance R 2 .
If power delivered in both cases is then the internal resistance of the cell is
R1 R - R2 R + R2
(1) R1R 2 (2) (3) 1 (4) 1
R2 2 2
node06\B0BA-BB\Kota\Board Material\Physics\CBSE\Booklet\Class XII\English\Term-I\3-Current Electricity

8. If the length of potentiometer wire is increased, then the length of the previously obtained
balance point will
(1) Increase (2) Decrease (3) Remain unchanged (4) Become two times
9. For comparing the e.m.f.'s of two cells with a potentiometer, a standard cell is used to develop a
potential gradient along the wires. Which of the following possibilities would make the
experiment unsuccessful
(1) The e.m.f. of the standard cell is larger than the E e.m.f.'s of the two cells
(2) The diameter of the wires is the same and uniform throughout
(3) The number of wires is ten
(4) The e.m.f. of the standard cell is smaller than the e.m.f.'s of the two cells

E
56 Physics
10. A current passes through a wire of non-uniform cross section. Which of the following quantities
are independent of the cross section :
(1) the charge crossing (2) drift velocity (3) current density (4) free electron density
11. Two cells of equal e.m.f. and of internal resistances r1 and r2 (r1 > r2 ) are connected in series. On
connecting this combination to an external resistance R, it is observed that the potential
difference across the first cell becomes zero. The value of R will be
r +r r -r
(1) r1 + r2 (2) r1 - r2 (3) 1 2 (4) 1 2
2 2
12. In the adjoining circuit diagram each resistance is of 10 W. The current in the arm AD will be :
E

B F i

i
A C

2i 3i 4i i
(1) (2) (3) (4)
5 5 5 5
ASSERTION & REASON QUESTIONS (13 to 17)
In the following questions an Assertion (A) is followed by a corresponding Reason (R). Use the
following keys to choose the appropriate answer.
(1) Both (A) and (R) are true, (R) is the correct explanation of (A).
(2) Both (A) and (R) are true, (R) is not the correct explanation of (A).
(3) (A) is true; (R) is false.
(4) (A) is false; (R) is false.
13. Assertion : The drift velocity of electrons in a metallic wire will decrease, if the temperature of
the wire is increased.
Reason : On increasing temperature, conductance of metallic wire decreases.
14. Assertion : A potentiometer of longer length is used for accurate measurement.
Reason : The potential gradient for a potentiometer of longer length with a given source of emf
becomes small.
15. Assertion : The average thermal velocity of the electrons in a conductor is zero.
node06\B0BA-BB\Kota\Board Material\Physics\CBSE\Booklet\Class XII\English\Term-I\3-Current Electricity

Reason : Direction of motion of electrons are randomly oriented.


16. Assertion : A current flow in a conductor only when there is an electric field within the
conductor.
Reason : The drift velocity of electron in presence of electric field decreases.
17. Assertion : Material used in the construction of a standard resistance is constantan or maganin.
Reason : Temperature coefficient of constantan (maganin) is very small.

ANSWER KEY
1. (2) 2. (2) 3. (1) 4. (3) 5. (2) 6. (2) 7. (1) 8. (1)
9. (4) 10. (4) 11. (2) 12. (1) 13. (2) 14. (1) 15. (1) 16. (3)
17. (1)

E
Physics 57

UNIT – III : MAGNETIC EFFECT OF CURRENT AND MAGNETISM


CHAPTER – 4 : MOVING CHARGES AND MAGNETISM

1. BIOT-SAVART LAW (BSL)


According to the Biot Savart Law, the magnitude of magnetic field dB is proportional to the
current I, the element length d and inversely proportional to the square of the distance r. Its
direction is perpendicular to the plane containing d and r .
Field at point P due to current element

µ0 Id sin  

dB   
4 r 2

0
 107 Tm / A  

4
0 = permeability of free space (air/vaccum)
Vector form :-
µ0 Id  r µ0 Id  rˆ
dB  
4 r3 4 r 2

2. APPLICATION OF BIOT SAVART LAW


(a) Finite length wire :
µI
BP  0 (sin 1  sin 2 ) 
4d 

(b) Infinite length wire:


µI
BP  0
2d 

(c) Magnetic field at centre of current carrying circular loop :


µ0 I
B0 
2R
Magnetic field at centre of current carrying circular coil (N 1)
µ0 NI
B0  , where N  number of turns
2R

E
58 Physics
(d) Magnetic field at an axial point of current carrying
circular coil :
µ0 NIR2
Bx 
2(x2  R2 )3/2 

R
[where sin  = ]
x  R2
2

Bx – x curve for circular coil

3. AMPERE'S CIRCUITAL LAW (ACL)


It states that line integral of magnetic field is equal to 0 times the total current passing through
the surface

 B ·d 0 I

4. APPLICATION OF ACL
(A) Field at a axial point of solenoid :
0 nI
For finite length :- BP  (cos 1  cos 2 )
2
Angle 1 and 2 both measured in same sense from the axis of the solenoid to end vectors.
For Infinite length :- Bin  0 ni
N Total turn
n  Bout  0
L Total length


E
Physics 59
(B) Field inside toroid :
B  0 nI , where n = N/2Rm, turn density
R1  R 2
mean radius R m 
2

(C) Solid cylindrical wire :

Point Result
µ0 I µ0 I
(1) r > R Bout = (2) r = R BS =
2r 2 R
µ Ir
(3) r < R Bin = 0 2 (4) r = 0 Baxis = 0
2R

5. MAGNETIC FORCE ON MOVING CHARGE IN MAGNETIC FIELD


F  v 
Vector from Fm  q(v  Bext ) Always  m 
Fm  Bext 

Magnitude form :
  

 

6. MOTION OF CHARGE IN UNIFORM MAGNETIC FIELD (CIRCULAR PATH )


mv2
(v  B,  =90°) qvB 
r

E
60 Physics
mv
(a) Radius of circular path : r  , where P = mv = 2mEK  2mqVacc
qB
2m
(b) Time period : T 
qB

(qBr)2
(c) Kinetic energy of charge : EK 
2m
Motion of charge in uniform magnetic field at any angle except 0° or 180° or 90°
(Helical path)
mvsin 
(a) Radius of helical path : r 
qB
2m
(b) Time period : T 
qB
2m
(c) Pitch of helix : P = (vcos) T, where T 
qB
Combined effect of E & B on moving charge
Electromagnetic or Lorentz force
FL  Fe  Fm    

7. MAGNETIC FORCE ON CURRENT CARRYING WIRE (OR CONDUCTOR)


(a) Straight wire :- Fm  I(L  Bext /uniform )

(b) Arbitrary wire :- Fm  I(L  Bext /uniform )


8. Magnetic force between two long parallel wires

F µ0 I1I 2 parallel currents  Attraction 


 N/m antiparallel currents  Repulsion 
2d  

E
Physics 61
9. MAGNETIC TORQUE ON A CLOSED CURRENT CIRCUIT
When a plane closed current circuit is placed in uniform magnetic field , it experience a zero net
force, but experience a torque given by   NI A B  M B  BINAsin  where A = area
vector outward from the face of the circuit where the current is anticlockwise, B = magnetic
induction of the uniform magnetic field.
M = magnetic moment = NI A
Unit of magnetic moment = Am2
Note : This expression can be used only if B is uniform.

10. MOVING COIL GALVANOMETER (MCG)


Principle : When current carrying coil is placed in a magnetic field it experiences a torque.
Scale

Pointer Permanent Magnet


Coil

N S
Pivot

Soft iron
Core
Uniform radial
magnetic field
It consists of a plane coil of many turns suspended in a radial magnetic field. When a current is
passed in the coil it experiences a torque which produces a twist in the suspension.
This deflection is directly proportional to the torque
  NIAB = K;

 K 
I   ;
 NAB 
K = elastic torsional constant of the suspension

K
I  C ; C 
NAB
C  Galvanometer constant

Current Sensitivity (CS)


It is defined as the deflection per unit current.
NAB
CS =  / I =
K

E
62 Physics
Voltage Sensitivity (VS)
It is defined as deflection per unit voltage.
 NAB 
VS =  / V =  / IR =  
 KR 
Galvanometer :
An instrument used to measure strength of current by measuring the deflection of coil due to
torque produced by a magnetic field.
  i 
A galvanometer can be converted into ammeter & voltmeter of varied scale as below.

Ammeter :
It is a modified form of suspended coil galvanometer, it is used to measure current. A shunt
(small resistance) is connected in parallel with galvanometer to convert into ammeter .

Ig Rg
S
I  Ig

where
Rg = galvanometer resistance
Ig = Maximum current that can flow through the galvanometer .
I = Maximum current that can be measured using the given ammeter .
An ideal ammeter has zero resistance.

Voltmeter :
A high resistance is put in series with galvanometer. It is used to measure potential difference.
V = Ig (Rg + R)
V
R  Rg
Ig
R  , Ideal voltmeter

E
Physics 63

PRACTICE QUESTIONS
CASE STUDY BASED QUESTIONS
Type -1
(Ampere's Circuital Law)
Ampere's law gives a method to calculate the magnetic field due to given current distribution.
According to it, the circulation  B.dl of the resultant magnitude field along a closed plane curve
is equal to 0 times the total current crossing the area bounded by the closed curve provided
the electric field inside the loop remains constant. Ampere's law is more useful under
certain symmetrical conditions. Consider one such case of a long. Straight wire with circular
cross-section (radius R) carrying current I uniformly distributed across this cross-section.
I

r
Line of
force

1. The magnetic field at radical distance r from the centre of the wire in the region r > R, is :
0 I I  IR 2
0 Ir 2
(1) (2) 0 (3) 0 (4)
2r 2 R 2r 2R
2. The magnetic field at a distance r in the region r < R is :
0 I  Ir 2
I  Ir
(1) (2) 0 2 (3) 0 (4) 0 2
2r 2R 2r 2R
3. A long straight wire of a circular cross section (radius a) carries a steady current I and the current
I is uniformly distributed across this cross-section. Which of the following plots represents the
variation of magnitude of magnetic field B with distance r from the centre of the wire?
B B B B

(1) (2) (3) (4)


O a r O a r O a r O a r

4. A long straight wire of radius R carries a steady current I. The current is uniformly distributed
across its cross-section. The ratio of magnitude field at R/2 and 2R is :
1 1
(1) (2) 2 (3) (4) 1
2 4
5. A current I flows along the length of an infinitely long straight thin walled pipe, then the
magnetic field is :
(1) uniform throughout the pipe but not zero
(2) zero only along the axis of the pipe
(3) zero at any point inside the pipe
(4) maximum at the centre and minimum at the edges

E
64 Physics
Type 2 : Assertion and Reason :-
Question 6 to 10:-Note:- In the following questions an Assertion (A) is followed by a corresponding
Reason (R). Use the following keys to choose the appropriate answer.
(1) Both (A) and (R) are true, (R) is the correct explanation of (A).
(2) Both (A) and (R) are true, (R) is not the correct explanation of (A).
(3) (A) is true; (R) is false.
(4) (A) is false; (R) is false.
6. Assertion : Magnetic field interacts with a moving charge and not with a stationary charge.
Reason : A moving charge produces a magnetic field.
7. Assertion : Free electrons always keep on moving in a conductor even then no magnetic force
act on them in magnetic field unless a current is passed through it.
Reason : The average velocity of free electron is zero.
8. Assertion : The coil is bound over the metallic frame in moving coil galvanometer.
Reason : The metallic frame help in making steady deflection without any oscillation.
9. Assertion : A circular loop carrying current lies in XY plane with its center at origin having a
magnetic flux in negative Z-axis.
Reason : Magnetic flux direction is independent of the direction of current in the conductor.
10. Assertion : The energy of charged particle moving in a uniform magnetic field does not change.
Reason : Work done by magnetic field on the charge is zero.

Type 3 : Multiple choice question (one correct answer) (Questions 11 to 21):-


11. A particle of charge 16  1018 coulomb moving with velocity 10 ms1 along the x-axis enters a
region where a magnetic field of induction B is along the y-axis, and an electric field of
magnitude 104 V/m is along the negative z-axis. If the charged particle continues moving along
the x-axis, the magnitude of B is
(1) 103 Wb / m2 (2) 103 Wb / m2 (3) 105 Wb / m2 (4) 1016 Wb / m2
12. Two coaxial solenoids 1 and 2 of the same length are set so that one is inside the other. The
number of turns per unit length are n1 and n 2 . The currents i1 and i2 are flowing in opposite
directions. The magnetic field inside the inner coil is zero. This is possible when
(1) i1  i2 and n1  n2 (2) i1  i2 and n1  n2 (3) i1  i2 and n1  n2 (4) None of these
13. What is shape of magnet in moving coil galvanometer to make the radial magnetic field
(1) Concave (2) Horse shoe magnet (3) Convex (4) None of these
14. Two long straight parallel conductors separated by a distance of 0.5m carry currents of 5A and
8A in the same direction. The force per unit length experienced by each other is
(1) 1.6  105 N (Attractive) (2) 1.6  105 N (repulsive)
(3) 16  105 N (Attractive) (4) 16  105 N (repulsive)

E
Physics 65
15. Two galvanometers A and B require 3mA and 5mA respectively to produce the same deflection
of 10 divisions. Then
(1) A is more sensitive than B (2) B is more sensitive than A
(3) A and B are equally sensitive (4) Sensitiveness of B is 5/3 times that of A
16. A particle of mass 0.6 g and having charge of 25 nC is moving horizontally with a uniform
velocity 1.2  104ms1 in a uniform magnetic field, then the value of the magnetic induction is
(g  10ms2 )
(1) Zero (2) 10 T (3) 20 T (4) 200 T
17. Two long parallel copper wires carry currents of 5A each in opposite directions. If the wires are
separated by a distance of 0.5m, then the force between the two wires is
(1) 105 N, Attractive (2) 105 N, repulsive
(3) 2  105 N, Attractive (4) 2  105 N, repulsive
18. A charge of 2.0 C moves with a speed of 3.0  106 ms1 along +ve X-axis. A magnetic field of
strength B  0.2 kˆ Tesla exists in space. What is the magnetic force (Fm ) on the charge
(1) Fm  1.2N Along + ve x  direction (2) Fm  1.2N Along – ve x  direction
(3) Fm  1.2N Along + ve y  direction (4) Fm  1.2N Along – ve y  direction
19. When a 12  resistor is connected with a moving coil galvanometer then its deflection reduces
from 50 divisions to 10 divisions. The resistance of the galvanometer is
(1) 24 (2) 36 (3) 48 (4) 60
20. A and B are two concentric circular conductors of center O and carrying currents I1 and i2 as
shown in the adjacent figure. If ratio of their radii is 1 : 2 and ratio of the flux densities at O due
to A and B is 1 : 3, then the value of i1 / i2 is
B

A
r2
r1
O
i1
i2

1 1 1 1
(1) (2) (3) (4)
6 4 3 2
21. Two long parallel wires P and Q are both perpendicular to the plane of the paper with distance
5 m between them. If P and Q carry current of 2.5 amp and 5 amp respectively in the same
direction, then the magnetic field at a point half way between the wires is
30 0 30 0
(1) (2) (3) (4)
2  2 2

ANSWER KEY
1. (1) 2. (4) 3. (1) 4. (4) 5. (3) 6. (1) 7. (1) 8. (1)
9. (3) 10. (1) 11. (2) 12. (3) 13. (1) 14. (1) 15. (1) 16. (3)
17. (2) 18. (3) 19. (3) 20. (1) 21. (4)

E
66 Physics
ASSIGNMENT-1
(Magnetic field, Biot-Savart law and its application)
1. A magnetic field can be produced by :
(1) A moving charge (2) A changing electric field
(3) None of these (4) Both of these
2. Two concentric circular coils of ten turns each are situated in the same plane. Their radii are
20 and 40 cm and they carry respectively 0.2 and 0.3 ampere current in opposite direction. The
magnetic field in weber / m2 at the center is :
35  7 5
(1) 0 (2) 0 (3) 0 (4)  0
4 80 80 4
3. A helium nucleus makes a full rotation in a circle of radius 0.8 meter in two seconds. The value
of the magnetic field B at the center of the circle will be
1019 2  1010
(1) (2) 1019 0 (3) 2  1010 0 (4)
0 0
4. In the figure shown there are two semicircles of radii r1 and r2 in which a current i is flowing.
The magnetic induction at the center O will be

r1

O
r2

0 i 0 i 0 i  r1  r2  0 i  r2  r1 
(1) (r  r ) (2) (r  r ) (3) (4)
r 1 2 4 1 2 4  r1r2  4  r1r2 
5. For the magnetic field to be maximum due to a small element of current carrying conductor at a
point, the angle between the element and the line joining the element to the given point must be
(1) 0° (2) 90° (3) 180° (4) 45°

ASSIGNMENT-2
(Ampere’s circuital law and its application)
6. A current i ampere flows along the inner conductor of a coaxial cable and returns along the outer
conductor of the cable, then the magnetic induction at any point outside the conductor at a
distance r meter from the axis is
 2i  2 i
(1)  (2) Zero (3) 0 (4) 0
4 r 4 r
7. A long solenoid has a radius a and number of turns per unit length is n. If it carries a current i,
then the magnetic field on its axis is directly proportional to
ni
(1) ani (2) ni (3) (4) n2 i
a

E
Physics 67
8. A solenoid is 1.0 meter long and it has 4250 turns. If a current of 5.0 ampere is flowing through
it, what is the magnetic field at its center [0  4  107 weber / amp  m]
(1) 5.4  102 weber / m2 (2) 2.7  102 weber / m2
(3) 1.35  102 weber / m2 (4) 0.675  102 weber / m2
9. A long solenoid of length L has a mean diameter D. It has n layers of windings of N turns each.
If it carries a current „i‟ the magnetic field at its center will be
(1) Proportional to D (2) Inversely proportional to D
(3) Independent of D (4) Proportional to L
10. An electric current passes through a long straight wire. At a distance 5 cm from the wire, The
magnetic field is B. The field at 20 cm from the wire would be
B B B B
(1) (2) (3) (4)
6 4 3 2
ASSIGNMENT-3
(Force on a moving charge and current-carrying conductor in uniform magnetic field)
11. Two thin long parallel wires separated by a distance „d‟ carry a current of „I ‟ A in the same
direction. They will
(1) Attract each other with a force of 0 i2 / (2d2 )
(2) Repel each other with a force of 0 i2 / (2d2 )
(3) Attract each other with a force of 0 i2 / (2d)
(4) Repel each other with a force of 0 i2 / (2d)
12. Which of the following statement is true
(1) The presence of a large magnetic flux through a coil maintains a current in the coil if the
circuit is continuous
(2) A coil of a metal wire kept stationary in a non-uniform magnetic field has an e.m.f. induced
in it
(3) A charged particle enters a region of uniform magnetic field at an angle of 85 to the
magnetic lines of force; the path of the particle is a circle
(4) There is no change in the energy of a charged particle moving in a magnetic field although a
magnetic force is acting on it
13. A current of 10 ampere is flowing in a wire of length 1.5 m. A force of 15 N acts on it when it is
placed in a uniform magnetic field of 2 tesla. The angle between the magnetic field and the
direction of the current is
(1) 30 (2) 45 (3) 60 (4) 90
14. A charged particle moves with velocity v in a uniform magnetic field B . The magnetic force
experienced by the particle is
(1) Always zero (2) Never zero
(3) Zero, if B and v are perpendicular (4) Zero, if B and v are parallel

E
68 Physics
15. A proton (mass  1.67  1027 kg and charge  1.6  1019 C) enters perpendicular to a magnetic
field of intensity 2 weber / m2 with a velocity 3.4  107 m / sec . The acceleration of the proton
should be
(1) 6.5  1015 m / sec2 (2) 6.5  1013 m / sec2 (3) 6.5  1011 m / sec2 (4) 6.5  109 m / sec2

ASSIGNMENT-4
(Torque on a current loop in uniform magnetic field, moving coil Galvanometer)
16. A 100 turn‟s coil shown in figure carries a current of 2 amp in a magnetic field B  0.2 Wb / m2 .
The torque acting on the coil is :
A B

N 10 cm S

D C
8 cm

(1) 0.32 Nm tending to rotate the side AD out of the page


(2) 0.32 Nm tending to rotate the side AD into the page
(3) 0.0032 Nm tending to rotate the side AD out of the page
(4) 0.0032 Nm tending to rotate the side AD into the page
17. In a moving coil galvanometer, the deflection of the coil  is related to the electrical current i by
the relation

(1) i  tan  (2) i   (3) i  2 (4) i  


18. A circular coil of diameter 7cm has 24 turns of wire carrying current of 0.75A. The magnetic
moment of the coil is
(1) 6.9 102 Amp  m2 (2) 2.3 102 Amp-m2 (3) 102 Amp  m2 (4) 103 Amp  m2
19. An ammeter of 5 ohm resistance can read 5 mA. If it is to be used to read 100 volts, how much
resistance is to be connected in series :
(1) 19.9995 (2) 199.995 (3) 1999.95 (4) 19995
20. A galvanometer having a resistance of 8 ohm is shunted by a wire of resistance 2 ohm. If the
total current is 1 amp, the part of it passing through the shunt will be
(1) 0.25 amp (2) 0.8 amp (3) 0.2 amp (4) 0.5 amp

ANSWER KEY
1. (4) 2. (4) 3. (2) 4. (3) 5. (2) 6. (2) 7. (2) 8. (2)
9. (3) 10. (2) 11. (3) 12. (4) 13. (1) 14. (4) 15. (1) 16. (1)
17. (2) 18. (1) 19. (4) 20. (2)

E
Physics 69

EXERCISE-1
(Magnetic field, Biot-Savart law and its application)
1. Vector form of Biot-savart's law is :
0  dl  r  0 2  dl  r 
(1) dB  i  (2) dB  i  
4  r  4  r 
0 2  dl  r  0  dl  r 
(3) dB  i   (4) dB  i 
4  r 2  4  r 3 
2. The magnetic field at the center of a circular coil of radius r carrying current I is B1 . The field at
B1
the center of another coil of radius 2 r carrying same current I is B2 . The ratio is :
B2
1
(1) (2) 1 (3) 2 (4) 4
2
3. The direction of magnetic lines of forces close to a straight conductor carrying current will be :

(1) Along the length of the conductor


(2) Radially outward
(3) Circular in a plane perpendicular to the conductor
(4) Helical
4. One Tesla is equal to :
(1) 107 gauss (2) 104 gauss (3) 104 gauss (4) 108 gauss
5. A coil having N turns carry a current I as shown in the figure. The magnetic field intensity at
point P is :

R
P
I

 0 NIR2  NI 0 NIR2
(1) (2) 0 (3) (4) Zero
2(R 2  x 2 )3/2 2R (R  x)2

(Ampere’s circuital law and its application)


6. If a long hollow copper pipe carries a direct current, the magnetic field associated with the
current will be :
(1) Only inside the pipe (2) Only outside the pipe
(3) Neither inside nor outside the pipe (4) Both inside and outside the pipe

E
70 Physics
7. Ampere's circuital law is given by :
(1)  H . dl   I
0 enc (2)  B. dl   I
0 enc (3)  B. dl   J
0 (4)  H . dl   J
0

8. A long solenoid is formed by winding 20 turns/ cm. The current necessary to produce a magnetic
field of 20 millitesla inside the solenoid will be approximately :
(1) 8.0 A (2) 4.0 A (3) 2.0 A (4) 1.0 A
9. A current of 1 ampere is passed through a straight wire of length 2.0 meters. The magnetic field
at a point in air at a distance of 3 meters from either end of wire and lying on the axis of wire will
be :
0 0 0
(1) (2) (3) (4) Zero
2 4 8

10. If the strength of the magnetic field produced 10 cm away from a infinitely long straight
conductor is 105 Weber / m2 , the value of the current flowing in the conductor will be :
(1) 5 ampere (2) 10 ampere (3) 500 ampere (4) 1000 ampere

(Force on a moving charge and current-carrying conductor in uniform magnetic field)


11. A uniform electric field and a uniform magnetic field are produced, pointed in the same
direction. An electron is projected with its velocity pointing in the same direction :
(1) The electron will turn to its right
(2) The electron will turn to its left
(3) The electron velocity will increase in magnitude
(4) The electron velocity will decrease in magnitude
12. An electron has mass 9  1031 kg and charge 1.6  1019 C is moving with a velocity of 106 m / s ,
enters a region where magnetic field exists. If it describes a circle of radius 0.10 m, the intensity
of magnetic field must be :
(1) 1.8  104 T (2) 5.6  105 T (3) 14.4  105 T (4) 1.3  106 T
13. Through two parallel wires A and B, 10 and 2 ampere of currents are passed respectively in
opposite direction. If the wire A is infinitely long and the length of the wire B is 2 m, the force on
the conductor B, which is situated at 10 cm distance from A will be :
(1) 8  105 N (2) 4  107 N (3) 4  105 N (4) 4  107 N
14. The unit of electric current “ampere” is the current which when flowing through each of two
parallel wires spaced 1 m apart in vacuum and of infinite length will give rise to a force between
them equal to :
(1) 1N / m (2) 2  107 N / m (3) 1  102 N / m (4) 4  107 N / m
15. Two free parallel wires carrying currents in opposite direction :
(1) Attract each other (2) Repel each other
(3) Neither attract nor repel (4) Get rotated to be perpendicular to each other

E
Physics 71
(Torque on a current loop in uniform magnetic field, moving coil galvanometer)
16. An electron moves with a constant speed v along a circle of radius r. Its magnetic moment will be
(e is the electron's charge) :
1
(1) evr (2) evr (3) r 2ev (4) 2rev
2
17. A moving coil galvanometer has N number of turns in a coil of effective area A, it carries a
current I. The magnetic field B is radial. The torque acting on the coil is :
(1) NA2 B2 I (2) NABI2 (3) N2 ABI (4) NABI
18. In order to increase the sensitivity of a moving coil galvanometer, one should decrease :
(1) The strength of its magnet (2) The torsional constant of its suspension
(3) The number of turns in its coil (4) The area of its coil
19. A circular coil of 20 turns and radius 10 cm is placed in uniform magnetic field of 0.10 T normal
to the plane of the coil. If the current in coil is 5 A, then the torque acting on the coil will be :
(1) 31.4 Nm (2) 3.14 Nm (3) 0.314 Nm (4) Zero
20. A galvanometer of 100  resistance gives full scale deflection when 10mA of current is passed.
To convert it into 10A range ammeter, the resistance of the shunt required will be
(1) 10  (2) 1 (3) 0.1 (4) 0.01

ANSWER KEY
1. (4) 2. (3) 3. (3) 4. (3) 5. (1) 6. (2) 7. (2) 8. (1)
9. (4) 10. (1) 11. (4) 12. (2) 13. (1) 14. (2) 15. (2) 16. (2)
17. (4) 18. (2) 19. (4) 20. (3)

E
72 Physics

CASE STUDY BASED QUESTIONS


(Moving Coil Galvanometer)
The galvanometer consists of a coil, with many turns, free to rotate about a fixed axis.
Its working is based on the fact that when a current carrying coil is placed in a magnetic field, it
experiences a torque. This torque tends to rotate the coil about its axis of suspension in such a
way that the magnetic flux passing through the coil is maximum.
A spring Sp provides a counter torque Kthat balances the magnetic torque NIBA, resulting in a
steady angular deflection.
Scale

Pointer Permanent Magnet


Coil

N S
Pivot

Soft iron
Core
Uniform radial
magnetic field
1. A moving coil galvanometer is an instrument which :
(1) is used to measure emf
(2) is used to measure potential difference
(3) is used to measure resistance
(4) is a deflection instrument which gives a deflection when a current flows through its coil
2. To make the field radial in a moving coil galvanometer :
(1) number of turns of coils is kept small (2) magnet is taken in the form of horse-shoe
(3) poles are of very strong magnets (4) poles are cylindrically cut
3. The deflection in a moving coil galvanometer is :
(1) directly proportional to torsional constant of spring
(2) directly proportional to the number of turns in the coil
(3) inversely proportional to the area of the coil
(4) inversely proportional to the current in the coil
4. In a moving coil galvanometer, having a coil of N-turns of area A and carrying current I is placed
in a rad field to strength B, to torque is given as :
(1) NA2B2I (2) NABI2 (3) N2ABI (4) NABI
5. To increase the current sensitivity of a moving coil galvanometer, we should decrease :
(1) strength of magnet (2) torsional constant of spring
(3) number of turns in coil (4) area of coil

ANSWER KEY
1. (4) 2. (2) 3. (2 ) 4. (4) 5. (2)

E
Physics 73

ASSERTION AND REASON QUESTIONS


Directions (Q. Nos. 1-5)
In the following questions an Assertion (A) is followed by a corresponding Reason (R). Use the
following keys to choose the appropriate answer.
(1) Both (A) and (R) are true, (R) is the correct explanation of (A).
(2) Both (A) and (R) are true, (R) is not the correct explanation of (A).
(3) (A) is true; (R) is false.
(4) (A) is false; (R) is false.
1. Assertion : A moving charged particle gets energy from magnetic field.
Reason : Magnetic force works on moving charged particle.
2. Assertion : Magnetic field lines can be entirely confined within the core of a toroid, but not
within a straight solenoid.
Reason : The magnetic field inside the solenoid is uniform.
3. Assertion : When a charged particle moves perpendicular to magnetic field then its kinetic
energy and momentum gets affected.
Reason : Magnetic force changes velocity of charged particle.
4. Assertion : The coil is wound over the metallic frame in moving coil galvanometer.
Reason : The metallic frame helps in making steady deflection without any oscillation.
5. Assertion : When a magnetic dipole is placed in a non-uniform magnetic field, only a torque acts
on the dipole.
Reason : Force would act on dipole if magnetic field is uniform.

ANSWER KEY
1. (4) 2. (2) 3. (4) 4. (1) 5. (4)

E
74 Physics

EXERCISE-2
1. Two straight horizontal parallel wires are carrying the same current in the same direction d is the
distance between the wires. You are provided with a small freely suspended magnetic needle. At
which of the following positions will the orientation of the needle be independent of the
magnitude of the current in the wires
(1) At a distance d / 2 from any of the wires
(2) At a distance d / 2 from any of the wires in the horizontal plane
(3) Anywhere on the circumference of a vertical circle of radius d and center halfway between
the wires
(4) At points halfway between the wires in the horizontal plane
2. The magnetic moment of a current carrying loop is 2.1  1025 amp  m2 . The magnetic field at a
point on its axis at a distance of 1Å is
(1) 4.2  102 weber / m2 (2) 4.2  103 weber / m2
(3) 4.2  104 weber / m2 (4) 4.2  105 weber / m2
3. A particle carrying a charge equal to 100 times the charge on an electron is rotating per second in
a circular path of radius 0.8 meter. The value of the magnetic field produced at the center will be
107
(1) (2) 1017 0 (3) 106 0 (4) 107  0
0
4. Magnetic fields at two points on the axis of a circular coil at a distance of 0.05m and 0.2m from
the center are in the ratio 8 : 1. The radius of the coil is
(1) 1.0 m (2) 0.1 m (3) 0.15 m (4) 0.2 m
5. Two similar coils are kept mutually perpendicular such that their centers coincide. At the center,
find the ratio of the magnetic field due to one coil and the resultant magnetic field by both coils,
if the same current is flown
(1) 1: 2 (2) 1 : 2 (3) 2 : 1 (4) 3 :1
6. A wire carrying current I and other carrying 2I in the same direction produces a magnetic field B
at the mid-point. What will be the field when 2I wire is switched off
(1) B / 2 (2) 2B (3) B (4) 4B
7. A proton (mass m and charge +e) and an   particle (mass 4m and charge +2e) are projected
with the same kinetic energy at right angles to the uniform magnetic field. Which one of the
following statements will be true
(1) The   particle will be bent in a circular path with a small radius that for the proton
(2) The radius of the path of the   particle will be greater than that of the proton
(3) The   particle and the proton will be bent in a circular path with the same radius
(4) The   particle and the proton will go through the field in a straight line

E
Physics 75
8. A triangular loop of side l carries a current I. It is placed in a magnetic field B such that the plane
of the loop is in the direction of B. The torque on the loop is

3 2 2 3
(1) Zero (2) IBl (3) Il B (4) IBl 2
2 4
9. The resistance of 1 A ammeter is 0.018 . To convert it into 10A ammeter, the shunt resistance
required will be
(1) 0.18 (2) 0.0018 (3) 0.002  (4) 0.12 
10. A circular loop has a radius of 5 cm and it is carrying a current of 0.1 amp. Its magnetic moment
is :
(1) 1.32  104 amp  m2 (2) 2.62  104 amp-m2

(3) 5.25  104 amp-m2 (4) 7.85  104 amp-m2


11. A moving coil galvanometer is converted into an ammeter reading upto 0.03A by connecting a
shunt of resistance 4r across it and into an ammeter reading upto 0.06 A when a shunt of
resistance r is connected across it. What is the maximum current which can be sent through this
galvanometer if no shunt is used
(1) 0.01A (2) 0.02 A (3) 0.03A (4) 0.04 A
12. A strong magnetic field is applied on a stationary electron. Then the electron :
(1) moves in the direction of the field
(2) remains stationary
(3) moves perpendicular to the direction of the field
(4) moves opposite to the direction of the field

ASSERTION & REASON QUESTIONS (13 to 17 )


In the following questions an Assertion (A) is followed by a corresponding Reason (R). Use the
following keys to choose the appropriate answer.
(1) Both (A) and (R) are true, (R) is the correct explanation of (A).
(2) Both (A) and (R) are true, (R) is not the correct explanation of (A).
(3) (A) is true; (R) is false.
(4) (A) is false; (R) is false.
13. Assertion : A magnetic field exert a force on a moving charge in the same direction as the
direction of field itself.
Reason : The direction of force is given by Fleming's left hand rule.

E
76 Physics
14. Assertion : When the observation point lies along the length of the current element, magnetic
field is zero.
Reason : Magnetic field close to current element is zero.
15. Assertion : A solenoid tends to expand, when a current passes through it.
Reason : Two straight parallel metallic wires carrying current in same direction in same
direction repel each other.
16. Assertion : A charge, whether stationary or in motion produces a magnetic field around it.
Reason : Moving charges produce only electric field in the surrounding space.
17. Assertion : The kinetic energy of a charged particle moving in a uniform magnetic field does not
change.
Reason : In a uniform magnetic field no force acts on the charge particle.

ANSWER KEY
1. (4) 2. (1) 3. (2) 4. (2) 5. (1) 6. (3) 7. (3) 8. (4)
9. (3) 10. (4) 11. (2) 12. (2) 13. (2) 14. (3) 15. (4) 16. (4)
17. (3)

E
Physics 77

CHAPTER – 5
MAGNETISM AND MATTER
1. MAGNETIC FIELD LINES
The magnetic field lines are a visual and intuitive realisation of the magnetic field.
The magnetic field line of a magnet form continuous close loops and tangent to the field line at a
given point represents the direction of the net magnetic field at the point.

2. MAGNETIC MOMENT OF REVOLVING ELECTRON


e
If an electron is revolving with T as a time period then equivalent current is given I 
T
eVr
magnetic moment is given as l = IA = Ir2 =
2
eL
  L = angular momentum
2me
NOTE: The ratio of magnetic moment to angular momentum called gyromagnetic ratio.
Bohr magnetron = 9.27 × 10–24 Am2

3. GAUSS'S LAW FOR MAGNETISM


It states that the net magnetic flux through any closed surface is zero B   B· S  0

4. EARTH'S MAGNETISM
Nm
11.3º

Nm
Magnetic Equator
Geographic Equator
Sm

Sm

(a) Imaginary vertical plane passing through the magnetic North - South poles at that place. This
plane is called the MAGNETIC MERIDIAN. The Earth's Magnetic poles are opposite to the
geometric poles i.e. at earth's north pole, its geomagnetic south pole is situated and vice versa.
(b) On the magnetic meridian, the magnetic induction vector of the earth at any point, generally
inclined to the horizontal at an angle called the MAGNETIC DIP at that place , such that B = total
magnetic induction of the earth at that point.
Bv = the vertical component of B in the magnetic meridian plane = B sin 
BH = the horizontal component of B in the magnetic meridian plane = B cos .
Bv
 tan 
BH
(c) At a given place on the surface of the earth , the magnetic meridian and the geographic
meridian may not coincide. The angle between them is called Declination at that place.

E
78 Physics
5. MAGNETIC PROPERTIES OF MATERIALS
  Intensity of magnetisation I = M/V
  Magnetic induction B = µH = µ0(H + I)

B
  Magnetic permeability µ=
H
I
  Magnetic susceptibility m= = µr – 1
H

6. IMPORTANT POINTS
(i) A charged particle moves perpendicular to magnetic field, its kinetic energy will remain constant
but momentum changes because magnetic force acts perpendicular to velocity of particle.
(ii) Magnetic force between two charges is generally much smaller than the electric force between
them because speeds of charges are much smaller than the free space speed of light.
Fmagnetic v2

Felectric c2

E
Physics 79

PRACTICE QUESTIONS
CASE STUDY BASED QUESTIONS
Type -1
(Elements of the Earth's Magnetic Field)
The earth magnetic field at a point on its surface is usually characterised by three quantities:
(a) declination (b) inclination or dip and (c) horizontal component of the field. These are known
as the elements of the earth's magnetic field. At a place, angle between geographic meridian and
magnetic meridian is defined as magnetic declination, whereas angle made by the Earth's
magnetic field with the horizontal in magnetic meridian is known as magnetic dip.
BH 
O

B BV
Magnetic
meridian
Geographic meridian
1
1. In a certain place, the horizontal component of magnetic field is times the vertical
3
component. The angle of dip at this place is :
(1) zero (2) /3 (3) /2 (4) /6
2. The angle between the true geographic north and the north shown by a compass needle is called
as :
(1) inclination (2) magnetic declination
(3) angle of meridian (4) magnetic pole.
3. The angles of dip at the poles and the equator respectively are :
(1) 30°, 60° (2) 0°, 90° (3) 45°, 90° (4) 90°, 0°
4. A compass needle which is allowed to move in a horizontal plane is taken to a magnetic pole. It :
(1) will become rigid showing no movement (2) will stay in any position
(3) will stay in north-south direction only (4) will stay in east-west direction only.
5. Select the correct statement from the following :
(1) The magnetic dip is zero at the centre of the earth.
(2) Magnetic dip decreases as we move away from the equator towards the magnetic pole.
(3) Magnetic dip increases as we move away from the equator towards the magnetic pole.
(4) Magnetic dip does not vary from place to place.

Type 2 : Assertion and Reason :-


Question 6 to 10 :-Note:- In the following questions an Assertion (A) is followed by a corresponding
Reason (R). Use the following keys to choose the appropriate answer.
(1) Both (A) and (R) are true, (R) is the correct explanation of (A).
(2) Both (A) and (R) are true, (R) is not the correct explanation of (A).
(3) (A) is true; (R) is false.
(4) (A) is false; (R) is false.

E
80 Physics
6. Assertion : We cannot think of magnetic field configuration with three poles.
Reason : A bar magnet does exert a torque on itself due to its own field.
7. Assertion : The poles of magnet cannot be separated by breaking into two pieces.
Reason : The magnetic moment will be reduced to half when a magnet is broken into two equal
pieces.
8. Assertion : Basic difference between an electric line and magnetic line of force is that former is
discontinuous and the latter is continuous or endless.
Reason : Electric lines of forces exist inside a charged body but magnetic lines do exist inside a
magnet.
9. Assertion : Magnetic moment of an atom is due to both, the orbital motion and spin motion of
every electron.
Reason : An accelerated charged particle produces a magnetic field.
10. Assertion : When radius of circular loop carrying current is doubled, its magnetic moment
becomes four times.
Reason : Magnetic moment of coil depends on area of the loop.
Type 3 : Multiple choice question (one correct answer) (Questions 11 to 18):-
11. A compass needle which is allowed to move in a horizontal plane is taken to a geomagnetic pole.
It :
(1) will stay in north–south direction only (2) will stay in east–west direction only
(3) will become rigid showing no movement (4) will stay in any position
12. A magnetic field exerts no force on :
(1) a magnet (2) an unmagnetised iron bar
(3) a moving charge (4) a charge at rest
13. Hysteresis is the phenomenon of lagging of :
(1) I behind B (2) B behind I (3) I and B behind H (4) H behind I
14. At a certain place, the angle of dip is 30° and the horizontal component of earth's magnetic field
is 0.50 orested. The earth's total magnetic field (in orested) is :
1 1
(1) 3 (2) 1 (3) (4)
3 2
15. A magnetic needle suspended freely oriented itself :
(1) in a definite direction (2) in no direction
(3) upward (4) downward
16. The magnetic moment for solenoid and corresponding bar magnet is :
(1) equal for both (2) more for solenoid
(3) more for bar magnet (4) none of these
17. The net magnetic flux through any closed surface, kept in a magnetic field is :
 4 0
(1) zero (2) 0 (3) 40 (4)
4 
18. Relative permeability of iron is 5500, then its magnetic susceptibility will be
(1) 5500  107 (2) 5500  10–7 (3) 5501 (4) 5499
ANSWER KEY
1. (2) 2. (2) 3. (4) 4. (2) 5. (3) 6. (1) 7. (1) 8. (4)
9. (2) 10. (1) 11. (4) 12. (4) 13. (3) 14. (1) 15. (1) 16. (1)
17. (1) 18. (4)

E
Physics 81
ASSIGNMENT-1
(Bar magnet, Magnetic dipole moment, Magnetic field lines)
1. A magnet of magnetic moment M and pole strength m is divided in two equal parts, then
magnetic moment of each part will be :
(1) M (2) M / 2 (3) M / 4 (4) 2M
2. A uniform magnetic field, parallel to the plane of the paper existed in space initially directed
from left to right. When a bar of soft iron is placed in the field parallel to it, the lines of force
passing through it will be represented by :

(A) (B) (C) (D)


(1) Figure (A) (2) Figure (B) (3) Figure (C) (4) Figure (D)
3. If the angular momentum of an electron is J then the magnitude of the magnetic moment will be:
eJ eJ 2m
(1) (2) (3) eJ 2m (4)
m 2m eJ
4. The magnetic moment of a magnet of length 10 cm and pole strength 4.0 Am will be :
(1) 0.4 Am2 (2) 1.6 Am2 (3) 20 Am2 (4) 8.0 Am2
5. The ratio of the orbital dipole moment to the orbital angular moment is given by :
e e e e
(1) (2)  (3) (4) 
M m 2M 2M

ASSIGNMENT-2
(Earth’s magnetic field, Magnetic elements)
6. When a piece of a ferromagnetic substance is put in a uniform magnetic field, the flux density
inside it is four times the flux density away from the piece. The magnetic permeability of the
material is :
(1) 1 (2) 2 (3) 3 (4) 4
7. Earth's magnetic field always has a horizontal component except at or Horizontal component of
earth's magnetic field remains zero at
(1) Equator (2) Magnetic poles (3) A latitude of 60o (4) An altitude of 60o
8. At which place, earth's magnetism become horizontal :
(1) Magnetic pole (2) Geographical pole (3) Magnetic meridian (4) Magnetic equator
9. For an isotropic medium B, , H and M are related as (where B, 0 , H and M have their usual
meaning in the context of magnetic material :
(1) (B  M)  0H (2) M  0 (H  M) (3) H  0 (H  M) (4) B  o (H  M)
10. At a certain place, the horizontal component of earth's magnetic field is 3 times the vertical
component. The angle of dip at that place is :
(1) 60 (2) 45 (3) 90 (4) 30°

ANSWER KEY
1. (2) 2. (2) 3. (2) 4. (1) 5. (4) 6. (4) 7. (2) 8. (4)
9. (4) 10. (4)

E
82 Physics

EXERCISE-1
(Bar magnet, Magnetic dipole moment, Magnetic field lines)
1. A magnet is placed in iron powder and then taken out, then maximum iron powder is at :
(1) Some away from North Pole (2) Some away from South Pole
(3) The middle of the magnet (4) The end of the magnet
2. Two identical thin bar magnets each of length l and pole strength m are placed at right angle to
each other with north pole of one touching south pole of the other. Magnetic moment of the
system is :
1
(1) ml (2) 2ml (3) 2ml (4) ml
2
3. A long magnet is cut in two parts in such a way that the ratio of their lengths is 2 : 1. The ratio of
pole strengths of both the section is :
(1) Equal (2) In the ratio of 2: 1 (3) In the ratio of 1: 2 (4) In the ratio of 4: 1
4. Force between two unit pole strength placed at a distance of one meter is :
107
(1) 1 N (2) N (3) 107 N (4) 4  107 N
4
5. Bohr Magnetron is given by :
eh eh eh eh
(1) (2) (3) (4)
2M 2M 4M 4M

(Earth’s magnetic field, Magnetic elements)


6. At magnetic poles of earth, angle of dip is :
(1) Zero (2) 45o (3) 90o (4) 180o
7. The angle between the magnetic meridian and geographical meridian is called :
(1) Angle of dip (2) Angle of declination
(3) Magnetic moment (4) Power of magnetic field
8. At a certain place the angle of dip is 30° and the horizontal component of earth's magnetic field is
0.50 Oersted. The earth's total magnetic field is
1 1
(1) 3 (2) 1 (3) (4)
3 2
9. The magnetic susceptibility is
I B M M
(1)   (2)   (3)   (4)  
H H V H
10. At a place, if the earth's horizontal and vertical components of magnetic fields are equal, then the
angle of dip will be
(1) 30 (2) 90 (3) 45 (4) 0°

ANSWER KEY
1. (4) 2. (3) 3. (1) 4. (3) 5. (4) 6. (3) 7. (2) 8. (3)
9. (1) 10. (3)

E
Physics 83

CASE STUDY BASED QUESTIONS


(Gauss's Law for Magnetism)
By analogy of Gauss's law off electrostatics, we can write gauss's law of magnetism as

 B.ds   m0 inside where  B.ds is the magnetic flux and minside is the net pole strength inside the
closed surface.
We do not have an isolated magnetic pole in nature. At least none has been found to exist till
date. The smallest unit of the source of magnetic field is magnetic dipole where the net magnetic
pole is zero. Hence, then net magnetic pole enclosed by any closed surface is always zero.
Correspondingly, the flux of the magnetic field though any closed surface is zero.

S N

1. The integral format Gauss's law is given by :


(1)  B . dA  0 (2)  B . dA  0 (3)  B . dA   0 (4)  B . dA   M
0

2. The net magnetic flux through any closed surface, kept in a magnetic field is :
0 4 0
(1) zero (2) (3) 40 (4)
4 
3. A closed surface S encloses a magnetic dipole of magnetic moment 2ml. The magnetic flux
emerging from the surface is :
2m
(1) 0m (2) zero (3) 20m (4)
0
4. Which of the following is not a consequence of Gass's law?
(1) The magnetic poles always exist as unlike pairs of equal strength.
(2) If several magnetic lines of force enter in a closed surface, then an equal number of lines of
force must leave that surface.
(3) These are abundant sources or sinks of the magnetic field inside a closed surface.
(4) Isolated magnetic poles do not exist.
5. The surface integral of a magnetic field over a surface :
(1) is proportional to mass enclosed (2) is proportional to charge enclosed
(3) is zero (4) equal to its magnetic flux through that surface

ANSWER KEY
1. (2) 2. (1) 3. (2) 4. (3) 5. (3)

E
84 Physics
ASSERTION AND REASON QUESTIONS
Directions (Q. Nos. 1-5)

In the following questions an Assertion (A) is followed by a corresponding Reason (R). Use the

following keys to choose the appropriate answer.

(1) Both (A) and (R) are true, (R) is the correct explanation of (A).

(2) Both (A) and (R) are true, (R) is not the correct explanation of (A).

(3) (A) is true; (R) is false.

(4) (A) is false; (R) is false.

1. Assertion : Magnetic moment of an atom is due to both, the orbital motion and spin motion of

every electron.

Reason : A charged particle produces a magnetic field.

2. Assertion : The earth's magnetic field is mainly due to revolving of earth.

Reason : At a high temperature magnetic losses its magnetic property or magnetism.

3. Assertion : The ends of a magnetic suspended freely point out always along north-south.

Reason : Earth behaves as a huge magnet.

4. Assertion : A compass needle when placed on the magnetic north pole of the earth rotates in

vertical direction.

Reason : The earth has only horizontal component of its magnetic field at the north poles.

5. Assertion : At neutral point, a compass needle point out in any arbitrary direction.

Reason : Magnetic field of earth is balanced by field due to magnets at the neutral points.

ANSWER KEY
1. (3) 2. (2) 3. (1) 4. (4) 5. (1)

E
Physics 85

EXERCISE-2

1. Two identical magnets each of moment M are kept at an angle of 60°. Such that like poles are
touching each other. The magnetic moment of the combination will be :
(1) M (2) 2M (3) 2M (4) 3M
2. A dip needle in a plane perpendicular to the magnetic meridian will remain :
(1) vertical (2) horizontal
(3) in any direction (4) at an angle of dip to the horizontal
3. A sensitive instrument can be shielded very effectively from outside fields by placing it inside a
box of :
(1) teak wood (2) plastic material
(3) soft iron of high permeability (4) a metal of high conductivity
4. The area enclosed by a hysteresis loop is a measure of :
(1) retentivity (2) susceptibility (3) permeability (4) energy loss per cycle
5. The temperature at and above which a ferromagnetic, material becomes paramagnetic is called :
(1) Critical temperature (2) Inversion temperature
(3) Curie temperature (4) Debye temperature
6. A magnet of magnetic moment M is cut into two equal parts. The two parts are placed
perpendicular to each other so that their north poles touch other. The resultant magnetic moment
is :
M M
(1) 2M (2) (3) 3M (4)
2 3
7. The magnetic lines of force inside a bar magnet :
(1) Do not exist
(2) Are from N–pole to S–pole of the magnet
(3) Are from S–pole to N–pole of the magnet
(4) Depend upon the area of cross–section of the bar magnet
8. The magnetic moment () of a revolving electron around the nucleus varies with principal
quantum number n as :
1 1
(1)  n (2)   (3)  n2 (4)   2
n n
9. The magnetic dipole moment of a current loop is independent of :
(1) Magnetic field in which it is lying (2) Number of turns
(3) Area of the loop (4) Current in the loop
10. Which one of the following in correct? The magnetic needle free to rotate in horizontal plane
stop in :
(1) the geographical meridian (2) the magnetic meridian
(3) the direction of dip (4) any direction other than the above three
11. A bar magnet is equivalent to a current–carrying :
(1) Toroid (2) Solenoid (3) Circular coil (4) Straight conductor

E
86 Physics
12. The earth's magnetic field always has a horizontal component except at :
(1) equator (2) Magnetic pole (3) A latitude of 60° (4) A latitude of 50°
13. At a place the angle of dip is 30°, if the horizontal component of the earth's magnetic field is H,
then the total field intensity will be given by :
H 2H
(1) (2) (3) H 2 (4) H 3
2 3

ASSERTION & REASON QUESTIONS (14 to 18)


In the following questions an Assertion (A) is followed by a corresponding Reason (R). Use the
following keys to choose the appropriate answer.
(1) Both (A) and (R) are true, (R) is the correct explanation of (A).
(2) Both (A) and (R) are true, (R) is not the correct explanation of (A).
(3) (A) is true; (R) is false.
(4) (A) is false; (R) is false.
14. Assertion : Magnetic moment is measured in joule/tesla or amp m2.
Reason : Joule/tesla is equivalent to amp m2.
15. Assertion : Earth's magnetic field does not affect the working of a moving coil galvanometer.
Reason : The earth's magnetic field is quite weak as compared to magnetic field produced in the
moving coil galvanometer.
16. Assertion : Angle between total magnetic field & horizontal component of earth magnetic field
is called angle of displacement.
Reason : When the horizontal and vertical components of the earth's magnetic field are equal,
the angle of dip is 60°.
17. Assertion : Gauss's Law of magnetism is different from that of for electrostatics.
Reason : Isolated monopoles are not known to exist.
18. Assertion : An atom behaves as magnetic dipole.
Reason : It is because an atom contains equal positive and negative charges.

ANSWER KEY
1. (4) 2. (1) 3. (3) 4. (4) 5. (3) 6. (2) 7. (3) 8. (2)
9. (1) 10. (4) 11. (2) 12. (2) 13. (2) 14. (1) 15. (1) 16. (1)
17. (1) 18. (3)

E
Physics 87
UNIT – IV : ELECTROMAGNETIC INDUCTION AND ALTERNATING CURRENTS
CHAPTER – 6 : ELECTROMAGNETIC INDUCTION
1. ELECTROMAGNETIC INDUCTION (EMI)
The phenomena in which emf is induced by varying magnetic field is called EMI.

2. MAGNETIC FLUX ()

 = NB.A  NBA cos 


SI unit : Weber (wb) or T-m2
Magnetic flux is scalar quantity

3. FARADAY'S LAW
It states that the magnitude of induced emf in a circuit is equal to the time rate of change of
magnetic flux through the circuit.
d
Induced emf   N
dt

4. LENZ LAW
It states that the direction of current induced in the loop is such that it oppose the cause due to
which it has produced.
It is in accordance with law of conservation of energy.
d
Induced emf = –N . Here negative sign is due to Lenz law
dt

Charge flown due to induced emf, Q 
R

5. MOTIONAL EMF
When a rod moves perpendicular to its length and perpendicular to magnetic field then emf
induces in rod called motional emf.


(v   B)

Motional induced emf e  B V

When a conducting disc or conducting rod is rotated about its axis to magnetic field then emf
B 2
induced between its centre and periphery is given by emf 
2

E
88 Physics

6. EDDY CURRENTS
When any metal (conductor of any shape) are subjected to changing magnetic flux, induced
current produced in the form of small loops (eddies) called eddy currents. It direction is given by
Lenz law.
Advantage / Application
(i) Magnetic breaking trains (ii) Electromagnetic damping
(iii) Induction motor (iv) Power meter
Disadvantage
Eddy current heat up the core of conductor and thus dissipated the energy.

7. INDUCTANCE
It is the ratio of flux linkage to current. It is equal to N / I.
Its scalar qty. SI unit Henry (H)
It depends on geometry of the conductor & intrinsic magnetic properties of material.

7.1 Self induction and Self Inductance :


Phenomena of induced emf due to change in its own current is called self induction.
Ldi
N   Li, emf  
dt

Self inductance (L) : Self inductance is numerically equal to the magnetic flux linked with coil
when a unit current flows through it.

Self inductance (L) for a solenoid :

0 N2 A
L  0 n 2 A

N = total number of turns; n = number of turns/length


Combination of inductors
1 1 1
Series L = L1+ L2 Parallel  
L L1 L2

E
Physics 89
7.2 Mutual induction & mutual inductance :
Phenomena of including emf in a coil due to change in current in another coil is known as mutual
induction.
di
N   Mi emf  M
dt

Mutual inductance (M) : Mutual inductance is numerically is equal to the magnetic flux linked
with one coil when a unit current flows through the other coil.
Mutual inductance between 2 solenoids

0 N1N2 A
M

Mutual induction of the pair of the coil, solenoids depends on their separation as well as their
relative orientation.
Relation between self inductance and mutual inductance.

M  k L1L2 M  L1L2

k = coupling factor k = 1 for ideal coil

8. ENERGY STORED IN INDUCTOR


1
U  Li2
2
B2
Energy density =
2µ0

9. AC GENERATOR
AC generator works on the principle of EMI

E
90 Physics
Coil Axle

Slip
rings
Alternating emf

Carbon
brushes

e  NBA sin t
e  e0 sin t

10. IMPORTANT POINTS

(i) Loop will repel the magnet

(ii) Loop will attract the magnet

(iii) An emf is induced in a closed loop where magnetic flux is varied. The induced electric
field is not conservative field because for induced electric field, the line integral  E.d
around a closed path is non-zero.
(iv) Acceleration of a magnet falling through a long solenoid decrease because the induced
current produced in a circuit always flows in such direction that it opposes the change or
the cause that produces it.

E
Physics 91
PRACTICE QUESTIONS

CASE STUDY BASED QUESTIONS


Type -1

(Eddy Currents and their Effects)

Currents can be induced not only in conducting coils, but also in conducting sheets or blocks.

Current is induced in solid metallic masses when the magnetic flux threading through them

changes. Such currents flow in the form of irregularly shaped loops throughout the body of the

metal. These currents look like eddies or whirlpools in water so they are known as eddy currents.

Eddy currents have both undesirable effects and practically useful applications. For example it

causes unnecessary heating and wastage of power in electric motors, dynamos and in the cores of

transformers.

1. The working of speedometers of trains is based on

(1) wattless currents (2) eddy currents

(3) alternating currents (4) pulsating currents

2. Identify the wrong statement.

(1) Eddy currents are produced in a steady magnetic field.

(2) Induction furnace uses eddy currents to produce heat.

(3) Eddy currents can be used to produce braking force in moving trains.

(4) Power meters work on the principle of eddy currents.

3. Which of the following is the best method to reduce eddy currents?

(1) Laminating core (2) Using thick wires

(3) By reducing hysteresis loss (4) None of these

4. The direction of eddy currents is given by

(1) Fleming's left hand rule (2) Biot-Savart law

(3) Lenz's law (4) Ampere-circuital law

5. Which of the following does not use the application of eddy current?

(1) Electric Power Metres (2) Induction furnace

(3) LED light (4) magnetic brakes in trains

E
92 Physics
Type 2 : Assertion and Reason :-
Question 6 to 10:-Note:- In the following questions an Assertion (A) is followed by a corresponding
Reason (R). Use the following keys to choose the appropriate answer.
(1) Both (A) and (R) are true, (R) is the correct explanation of (A).
(2) Both (A) and (R) are true, (R) is not the correct explanation of (A).
(3) (A) is true; (R) is false.
(4) (A) is false; (R) is false.
6. Assertion : Eddy currents is produced in any metallic conductor when magnetic flux is changed
around it.
Reason : Electric potential determines the flow of charge.
7. Assertion : Faraday’s laws are consequences of conservation of energy.
Reason : In a purely resistive ac circuit, the current lags behind the e.m.f. in phase.
8. Assertion : Only a change in magnetic flux will maintain an induced current the coil.
Reason : The presence of large magnetic flux through a coil maintains a current in the coil if the
circuit is continuous.
9. Assertion : Inductance coil are made of copper.
Reason : Induced current is more in wire having less resistance.
10. Assertion : Self-inductance is called the inertia of electricity.
Reason : Self-inductance is the phenomenon, according to which an opposing induced e.m.f. is
produced in a coil as a result of change in current or magnetic flux linked in the coil.

Type 3 : Multiple choice question (one correct answer) (Questions 11 to 20):-


11. A circular coil of 500 turns of wire has an enclosed area of 0.1m2 per turn. It is kept
perpendicular to a magnetic field of induction 0.2 T and rotated by 180° about a diameter
perpendicular to the field in 0.1 sec. How much charge will pass when the coil is connected to a
galvanometer with a combined resistance of 50 ohms :
(1) 0.2 C (2) 0.4 C (3) 2 C (4) 4 C
12. A magnetic field of 2  10– 2 T acts at right angles to a coil of area 100 cm2 with 50 turns. The
average emf induced in the coil is 0.1 V, when it is removed from the field in time t. The value of
t is :
(1) 0.1 sec (2) 0.01 sec (3) 1 sec (4) 20 sec
2
13. If a coil of 40 turns and area 4.0 cm is suddenly removed from a magnetic field, it is observed
that a charge of 2.0  10 4 C flows into the coil. If the resistance of the coil is 80 , the magnetic
flux density in Wb / m2 is :
(1) 0.5 (2) 1.0 (3) 1.5 (4) 2.0

E
Physics 93
14. A coil of area 80 square cm and 50 turns is rotating with 2000 revolutions per minute about an
axis perpendicular to a magnetic field of 0.05 Tesla. The maximum value of the e.m.f. developed
in it is :
10 4 2
(1) 200 volt (2) volt (3) volt (4) volt
3 3 3
15. A metal rod moves at a constant velocity in a direction perpendicular to its length. A constant
uniform magnetic field exists in space in a direction perpendicular to the rod as well as its
velocity. Select the correct statement(s) from the following :
(1) The entire rod is at the same electric potential
(2) There is an electric field in the rod
(3) The electric potential is highest at the center of the rod and decreases towards its ends
(4) The electric potential is lowest at the center of the rod and increases towards its ends
16. The back e.m.f. induced in a coil, when current changes from 1 ampere to zero in one
milli-second, is 4 volts, the self-inductance of the coil is :
(1) 1 H (2) 4 H (3) 10 3 H (4) 4  10 3 H
17. The coefficient of self-inductance of a solenoid is 0.18 mH. If a core of soft iron of relative
permeability 900 is inserted, then the coefficient of self-inductance will become nearly :
(1) 5.4 mH (2) 162 mH (3) 0.006 mH (4) 0.0002 mH
18. The unit of inductance is :
(1) Volt/ampere (2) Joule/ampere (3) Volt-sec/ampere (4) Volt-ampere/sec
19. Two coils have a mutual inductance 0.005 H. The current changes in the first coil according to
equation I  I0 sin  t , where I0  10A and  = 100  radian/sec. The maximum value of e.m.f.
in the second coil is :
(1) 2 (2) 5 (3)  (4) 4
20. If in a coil rate of change of area is 5 m2/millisecond and current become 1 amp from 2 amp in
2  103 sec. If magnitude of field is 1 tesla then self-inductance of the coil is :
(1) 2 H (2) 5 H (3) 20 H (4) 10 H

ANSWER KEY
1. (2) 2. (1) 3. (1) 4. (3) 5. (3) 6. (2) 7. (3) 8. (3)
9. (1) 10. (1) 11. (2) 12. (1) 13. (2) 14. (3) 15. (2) 16. (4)
17. (2) 18. (3) 19. (2) 20. (4)

E
94 Physics
ASSIGNMENT-1
(Electromagnetic induction, Faraday’s law, Lenz’s law)
1. The direction of induced e.m.f. during electromagnetic induction is given by
(1) Faraday's law (2) Lenz's law (3) Maxwell's law (4) Ampere's law
2. A solenoid is 1.5 m long and its inner diameter is 4.0 cm. It has three layers of windings of
1000 turns each and carries a current of 2.0 amperes. The magnetic flux for a cross-section of the
solenoid is nearly
(1) 2.5  10–7 weber (2) 6.31  10–6 weber (3) 5.2  10–5 weber (4) 4.1  10–5 weber
3. A magnetic field of 2  10– 2 T acts at right angles to a coil of area 100 cm2 with 50 turns. The
average emf induced in the coil is 0.1 V, when it is removed from the field in time t. The value of
t is
(1) 0.1 sec (2) 0.01 sec (3) 1 sec (4) 20 sec
4. The formula for induced e.m.f. in a coil due to change in magnetic flux through the coil is (here
A = area of the coil, B = magnetic field)
dB dA d d
(1) e  A. (2) e  B. (3) e   (A.B) (4) e   (A  B)
dt dt dt dt
5. If a copper ring is moved quickly towards south pole of a powerful stationary bar magnet, then
(1) Current flows through the copper ring (2) Voltage in the magnet increase
(3) Current flows in the magnet (4) Copper ring will get magnetized

ASSIGNMENT-2
(Induced parameters, Eddy currents)
6. In a coil of area 10 cm2 and 10 turns with a magnetic field directed perpendicular to the plane
and is changing at the rate of 108 gauss/second. The resistance of the coil is 20 ohm. The current
in the coil will be
(1) 5 amp (2) 0.5 amp (3) 0.05 amp (4) 5  108 amp
7. As shown in the figure, a magnet is moved with a fast speed towards a coil at rest. Due to this
induced electromotive force, induced current and induced charge in the coil is E, I and Q
respectively. If the speed of the magnet is doubled, the incorrect statement is
N S

(1) E Increases (2) I Increases (3) Q Remains same (4) Q Increases


8. A metal rod of length 2 m is rotating with an angular velocity of 100 rad/sec in a plane
perpendicular to a uniform magnetic field of 0.3 T. The potential difference between the ends of
the rod is
(1) 30 V (2) 40 V (3) 60 V (4) 600 V

E
Physics 95
9. The core of a transformer is laminated to reduce energy losses due to
(1) Eddy currents (2) Hysteresis
(3) Resistance in winding (4) None of these
10. Eddy currents are produced when
(1) A metal is kept in varying magnetic field
(2) A metal is kept in the steady magnetic field
(3) A circular coil is placed in a magnetic field
(4) Through a circular coil, current is passed

ASSIGNMENT-3
Self and mutual induction
11. If a change in current of 0.01 A in one coil produces a change in magnetic flux of 1.2  102 Wb
in the other coil, then the mutual inductance of the two coils in Henry is
(1) 0 (2) 0.5 (3) 1.2 (4) 3
12. When the number of turns in a coil is doubled without any change in the length of the coil, its
self-inductance becomes
(1) Four times (2) Doubled (3) Halved (4) Unchanged
13. Two coils are placed close to each other. The mutual inductance of the pair of coils depends upon
(1) The currents in the two coils
(2) The rates at which currents are changing in the two coils
(3) Relative position and orientation of the two coils
(4) The materials of the wires of the coils
14. In the figure magnetic energy stored in the coil is

2H

10 V 2

(1) Zero (2) Infinite (3) 25 joules (4) None of the above
15. In a transformer, the coefficient of mutual inductance between the primary and the secondary coil
is 0.2 henry. When the current changes by 5 ampere/second in the primary, the induced e.m.f. in
the secondary will be
(1) 5 V (2) 1 V (3) 25 V (4) 10 V

ANSWER KEY
1. (2) 2. (2) 3. (1) 4. (3) 5. (1) 6. (1) 7. (4) 8. (3)
9. (1) 10. (1) 11. (3) 12. (1) 13. (3) 14. (3) 15. (2)

E
96 Physics
EXERCISE-1
(Electromagnetic Induction, Faraday’s law, Lenz’s law)
1. In electromagnetic induction, the induced e.m.f. in a coil is independent of :
(1) Change in the flux (2) Time
(3) Resistance of the circuit (4) None of the above
2. A cylindrical bar magnet is kept along the axis of a circular coil. If the magnet is rotated about its
axis, then :
(1) A current will be induced in a coil
(2) No current will be induced in a coil
(3) Only an e.m.f. will be induced in the coil
(4) An e.m.f. and a current both will be induced in the coil
3. The magnetic flux linked with a coil is given by an equation  (in Weber’s) = 8t 2  3t  5 . The
induced e.m.f. in the coil at the fourth second will be :
(1) 16 units (2) 39 units (3) 67 units (4) 145 units
4. A coil having 500 square loops each of side 10 cm is placed normal to a magnetic flux which
increases at the rate of 1.0 tesla/second. The induced e.m.f. in volts is :
(1) 0.1 (2) 0.5 (3) 1 (4) 5
5. A coil of 100 turns and area 5 square centimeter is placed in a magnetic field B = 0.2 T. The
normal to the plane of the coil makes an angle of 60° with the direction of the magnetic field.
The magnetic flux linked with the coil is :
(1) 5  103Wb (2) 5 104 Wb (3) 102 Wb (4) 104 Wb

(Induced parameters, Eddy currents)


6. In electromagnetic induction, the induced charge in a coil is independent of :
(1) Change in the flux (2) Time
(3) Resistance in the circuit (4) None of the above
7. A metallic ring is attached with the wall of a room. When the north pole of a magnet is brought
near to it, the induced current in the ring will be :

S
a
N

(1) First clockwise then anticlockwise (2) In clockwise direction


(3) In anticlockwise direction (4) First anticlockwise then clockwise

E
Physics 97
8. In a circuit with a coil of resistance 2 ohms, the magnetic flux changes from 2.0 Wb to 10.0 Wb
in 0.2 second. The charge that flows in the coil during this time is :
(1) 5.0 coulomb (2) 4.0 coulomb (3) 1.0 coulomb (4) 0.8 coulomb
9. A 10 meter wire kept in east-west falling with velocity 5m/sec perpendicular to the field
0.3  104 Wb / m 2 . The induced e.m.f. across the terminal will be :
(1) 0.15 V (2) 1.5 mV (3) 1.5 V (4) 15.0 V
10. Which of the following is not an application of eddy currents :
(1) Induction furnace (2) Galvanometer damping
(3) Speedometer of automobiles (4) X-ray crystallography

(Self and Mutual Induction)


11. When the current in a coil changes from 8 ampere to 2 ampere in 3  102 second, the e.m.f.
induced in the coil is 2 volt . The self-inductance of the coil (in millihenry) is :
(1) 1 (2) 5 (3) 20 (4) 10
12. A coil and a bulb are connected in series with a AC source, a soft iron core is then inserted in the
coil. Then :
(1) Intensity of the bulb remains the same (2) Intensity of the bulb decreases
(3) Intensity of the bulb increases (4) The bulb ceases to glow
13. The energy stored in a 50 mH inductor carrying a current of 4 A will be :
(1) 0.4 J (2) 4.0 J (3) 0.8 J (4) 0.04 J
14. The inductance of a closed-packed coil of 400 turns is 8 mH. A current of 5 mA is passed
through it. The magnetic flux through each turn of the coil is :
1 1 1
(1)  Wb (2) 0Wb (3)  Wb (4) 0.4 0 Wb
4 0 2 3 0
15. The inductance of a solenoid 0.5 m long of cross-sectional area 20 cm2 and with 500 turns is :
(1) 12.5 mH (2) 1.25 mH (3) 15.0 mH (4) 0.12 mH

ANSWER KEY
1. (3) 2. (2) 3. (3) 4. (4) 5. (1) 6. (2) 7. (3) 8. (2)
9. (2) 10. (4) 11. (4) 12. (2) 13. (1) 14. (1) 15. (2)

E
98 Physics
CASE STUDY BASED QUESTIONS

(Self Induction)
When a current I flows through a coil, flux linked with it is  = LI, where L is a constant known
as self-inductance of the coil. Any change in current sets up an induced emf in the coil. Thus,
self-inductance of a coil is the induced emf set up in it when the current passing through it
changes at the unit rate. It is a measure of the opposition to the growth or the decay of current
flowing through the coil. Also, value of self-inductance depends on the number of turns in the
solenoid, its area of cross-section and the relative permeability of its core material.

1. The inductance in a coil play the same role as :


(1) inertia in mechanics (2) energy in mechanics
(3) momentum in mechanics (4) force in mechanics
2. The current of 2.5 A flows through a coil of inductance 5H. The magnetic flux linked with the
coil is :
(1) 0.5 Wb (2) 12.5 Wb (3) zero (4) 2 Wb
3. The inductance L of a solenoid depends upon its radius R as :
(1) L  R (2) L  1/R (3) L  R2 (4) L  R3
4. The unit of self-inductance is :
(1) weber/ampere (2) weber–1 ampere (3) ohm second (4) farad
5. The induced e.m.f. in a coil of 10 henry inductance in which current varies from 9 A to 4 A in
0.2 second is :
(1) 200 V (2) 250 V (3) 300 V (4) 350 V

ANSWER KEY
1. (1) 2. (2) 3. (3) 4. (1) 5. (2)

E
Physics 99
ASSERTION AND REASON QUESTIONS

Directions (Q. Nos. 1-5)

In the following questions an Assertion (A) is followed by a corresponding Reason (R). Use the

following keys to choose the appropriate answer.

(1) Both (A) and (R) are true, (R) is the correct explanation of (A).

(2) Both (A) and (R) are true, (R) is not the correct explanation of (A).

(3) (A) is true; (R) is false.

(4) (A) is false; (R) is false.

1. Assertion : When two coils are wound on each other, the mutual induction between the coils

maximum.

Reason : Mutual induction does not depend on the orientation of the coils.

2. Assertion : An induced emf is generated when magnet is withdrawn from the solenoid.

Reason : The relative motion between magnet and solenoid induces emf.

3. Assertion : An induced emf appears in any coil in which the current is changing.

Reason : Self induction phenomenon obey Faraday's law of induction.

4. Assertion : Changing magnetic flux can produced e.m.f.

Reason : Faraday established induced e.m.f. experimentally.

5. Assertion : Induced e.m.f. depends on number of turns and area of the coil.

Reason : Induced e.m.f. increases with increase in number of turns of coil.

ANSWER KEY
1. (3) 2. (1) 3. (2) 4. (2) 5. (2)

E
100 Physics

EXERCISE - 2
1. A magnet is brought towards a coil (i) speedly (ii) slowly then the induced e.m.f./induced charge
will be respectively :
(1) More in first case / More in first case (2) More in first case/Equal in both case
(3) Less in first case/More in second case (4) Less in first case/Equal in both case

2. A coil of area 100cm2 has 500 turns. Magnetic field of 0.1 weber / metre2 is perpendicular to the

coil. The field is reduced to zero in 0.1 second. The induced e.m.f. in the coil is :
(1) 1 V (2) 5 V (3) 50 V (4) Zero

3. A coil has 2000 turns and area of 70 cm2 . The magnetic field perpendicular to the plane of

the coil is 0.3Wb / m2 and takes 0.1 sec to rotate through 180o . The value of the induced e.m.f.
will be :
(1) 8.4V (2) 84V (3) 42V (4) 4.2V

4. The magnetic flux linked with coil, in weber is given by the equation,   5t 2  3t  16 . The

induced emf in the coil in the fourth second is :


(1) 10 V (2) 30 V (3) 45 V (4) 90 V
5. A rectangular coil of 300 turns has an average area of average area of 25 cm  10 cm. The coil

rotates with a speed of 50 cps in a uniform magnetic field of strength 4  10 2 T about an axis
perpendicular of the field. The peak value of the induced e.m.f. is (in volt) :
(1) 3000 (2) 300 (3) 30 (4) 3
6. A solenoid has 2000 turns wound over a length of 0.30 meter. The area of its cross-section is

1.2  103 m2 . Around its central section, a coil of 300 turns is wound. If an initial current of 2 A
in the solenoid is reversed in 0.25 sec, then the e.m.f. induced in the coil is :

(1) 6  104 V (2) 4.8  10 3 V (3) 6  102 V (4) 48 mV


7. Two coils of self-inductance L1 and L 2 are placed closer to each other so that total flux in one

coil is completely linked with other. If M is mutual inductance between them, then :

(1) M  L1L2 (2) M  L1 / L2 (3) M  L1L2 (4) M  (L1L2 )2

8. A coil of wire of a certain radius has 600 turns and a self-inductance of 108 mH. The
self-inductance of a 2nd similar coil of 500 turns will be :
(1) 74 mH (2) 75 mH (3) 76 mH (4) 77 mH

E
Physics 101
9. A circular coil of radius 5 cm has 500 turns of a wire. The approximate value of the coefficient of
self-induction of the coil will be :
(1) 25 millihenry (2) 25  103 millihenry

(3) 50  103 millihenry(4) 50  103 henry


10. The device that does not work on the principle of mutual induction is :
(1) Induction coil (2) Motor (3) Tesla coil (4) Transformer
11. Some magnetic flux is changed from a coil of resistance 10 ohm. As a result an induced current
is developed in it, which varies with time as shown in figure. The magnitude of change in flux
through the coil in webers is
i (amp)
4

t (sec)
0.1

(1) 2 (2) 4 (3) 6 (4) None of these

ASSERTION & REASON QUESTIONS (12 to 20)


In the following questions an Assertion (A) is followed by a corresponding Reason (R). Use the
following keys to choose the appropriate answer.
(1) Both (A) and (R) are true, (R) is the correct explanation of (A).
(2) Both (A) and (R) are true, (R) is not the correct explanation of (A).
(3) (A) is true; (R) is false.
(4) (A) is false; (R) is false.
12. Assertion : Inductance coil are made of copper.
Reason : Induced current is more in wire having less resistance.
13. Assertion : When number of turns in a coil doubled coefficient of self-inductance of the coil
becomes four times.
Reason : Coefficient of self-inductance is proportional to the square of number of turns.
14. Assertion : The reactance of a coil for direct current is 5 ohms. An alternating current is sent
through it. The reactance will remain same.
Reason : The reactance of a coil does not depend upon frequency of AC of current.
15. Assertion : Acceleration of a magnet falling through a copper ring decreases.
Reason : The induced current produced in a circuit always flow in such direction that it opposes
the change or the cause that produced it.

E
102 Physics
16. Assertion : An aircraft flies along the meridian the potential develops at the ends of its wings.

Reason : Whenever there is change in the magnetic flux e.m.f. induces.

17. Assertion : When two coils are wound on each other, the mutual induction between the coils is

maximum.

Reason : Mutual induction does not depend on the orientation of the coils.

18. Assertion : Acceleration of a magnet falling through a long solenoid decreases.

Reason : The induced current produced in a circuit always flow in such direction that it opposes

the change or the cause the produced it.

19. Assertion : A spark occurs between the poles of a switch when the switch is opened.

Reason : Current flowing in the conductor produces magnetic field.

20. Assertion : The induced emf in a conducting loop of wire will be non-zero when it rotates in a

uniform magnetic field.

Reason : The emf is induced due to change in magnetic flux.

ANSWER KEY
1. (2) 2. (2) 3. (2) 4. (1) 5. (3) 6. (4) 7. (3) 8. (2)
9. (1) 10. (3) 11. (1) 12. (1) 13. (1) 14. (4) 15. (1) 16. (1)
17. (3) 18. (1) 19. (2) 20. (1)

E
Physics 103
CHAPTER – 7
ALTERNATING CURRENT

1. Nature Wave–form
(voltage, current) : Sinusoidal
V  V0 sin t
I  I 0 sin t
 

2. ROOT MEAN SQUARE VALUE OF AC CURRENT (RMS)/EFFECTIVE CURRENT (Irms)


I0
Irms   0.707 I0 , I0 = peak value of current
2

3. AVERAGE OR MEAN VALUE


2I 0
I Avg   0.637 I 0

4. PHASORS AND PHASOR DIAGRAM


A phasor is a vector which rotates about the origin with angular speed . Phasor diagram shows
the phase relationship between voltage & current.

i
V
I
0

Phasor Diagram

5. AC VOLTAGE APPLIED TO A PURE RESISTANCE (R)

V  V0 sin t
V0
i sin t
R
Resistance = R

VR = iR
voltage and current are in same phase.

E
104 Physics
6. AC VOLTAGE APPLIED TO A PURE INDUCTOR (L)

V = V0 sin t
V0
i ( cos t)
L
Reactance XL = L

VL = iXL
voltage leads the current by /2

7. AC VOLTAGE APPLIED TO A PURE CAPACITOR (C)

V = V0 sin t
V0
i cos t
1/ C
1
Reactance XC 
C

VC = iXC
current leads voltage by /2

8. AC THROUGH LCR CIRCUIT

(a) If VL > VC XL > XC then


E
Physics 105
(b) If VC > VL XC > XL then
 

(c) If VC = VL XC = XL then V = VR , Z = R and  = 0

9. IMPEDANCE
Impedance = Z  R2  (XL  XC )2

10. POWER IN AC CIRCUIT


V = V0sin t
i = I0 sin(t + )
Power = Vrms irms cos = i2 R
Wattfull current = irms cos 

Wattless current = irms sin
Wattless power = vrms irms sin
 Where cos = Power factor

11. RESONANCE IN SERIES LCR CIRCUIT


At resonance
 XL = XC or VL = VC
V
 Z = R = min  i   max
R
 R 
 Power factor  cos    1
 Z 
 Angle (or phase difference) between v and i = 0°
 VR = VSource
1
Resonating frequency 0 
LC

   

XL 1 L f0
Sharpness of resonance / quality factor =  
R R C Band width

E
106 Physics

12. LC - OSCILLATION

d2Q Q
VC + VL = 0   L  0
dt 2 C
Q = Q0 cos t  i = –i0 sin t where i0 = Q0 
1
where   frequency of oscillation
LC

13. TRANSFORMER
It transfers electric power.
Works only for AC
Principle : Mutual induction
For ideal transformer

(1) Power loss = 0 efficiency = 100%

(2) Flux loss = 0

But practically Pout < Pin efficiency < 100%

VS NS ip is2 R
   turn ratio (efficiency)  100
VP NP is VP i p

Transmissions are done at high voltage and low current by using step up transformer.

E
Physics 107

14. IMPORTANT POINTS


 An alternating current of frequency 50 Hz becomes zero, 100 times in one second because
alternating current changes direction and becomes zero twice in a cycle.
 An alternating current cannot be used to conduct electrolysis because the ions due to their
inertia, cannot follow the changing electric field.
 Average value of AC is always defined over half cycle because average value of AC over a
complete cycle is always zero.
 AC current flows on the periphery of wire instead of flowing through total volume of wire.
This known as skin effect.
Note : Important points for LCR series circuit.


Vrms (Vrms )L (Vrms )C (Vrms )R
 I rms    
Z XL XC R

 Vrms  (Vrms )2R  [(Vrms )L  (Vrms )C ]2


| VL  VC | | X L  X C |
 tan = 
VR R
2
RVrms
 < P > = Vrms Irms cos  =
Z2

E
108 Physics

PRACTICE QUESTIONS
CASE STUDY BASED QUESTIONS
Type -1
(Average Power Associated with an Inductor and Capacitor)
The power averaged over on full cycle of a.c. is known as average power. It is also known as true
power.
VI
Pav  Vrms I rms cos   0 0 cos 
2
Root mean square or simply rms watts refer to continuous power.
A circuit containing a 5 H inductor 40 resistance and 80 F capacitor Resistance of 40in
series is connected to a 230 V, 50 Hz supply. The resistance of the circuit is negligible.

5H 80F
40

~
230 V, 50 Hz
1. The value of Peak current is :
(1) 6.5 A (2) 8.1 A (3) 12 A (4) 4 A
2. Find rms value :
(1) 6 A (2) 5.25 A (3) 5.75 A (4) 6.25 A
3. The average power transferred to inductor is :
(1) zero (2) 7 W (3) 2.5 W (4) 5 W
4. Potential drop across L-C combination is:
(1) 5 V (2) zero (3) 45 V (4) 150 V
5. What is the total average power absorbed by the circuit?
(1) zero (2) 4.5 W (3) 1.3 W (4) 8 W
Type 2 : Assertion and Reason :-
Question 6 to 10 :-Note:- In the following questions an Assertion (A) is followed by a corresponding
Reason (R). Use the following keys to choose the appropriate answer.
(1) Both (A) and (R) are true, (R) is the correct explanation of (A).
(2) Both (A) and (R) are true, (R) is not the correct explanation of (A).
(3) (A) is true; (R) is false.
(4) (A) is false; (R) is false.
6. Assertion : A transformer cannot work on dc supply.
Reason : dc changes neither in magnitude nor in direction.
7. Assertion : The alternating current lags behind the e.m.f. by a phase angle of  / 2 , when ac
flows through an inductor.
Reason : The inductive reactance increases as the frequency of ac source decreases.
8. Assertion : Average value of ac over a complete cycle is always zero.
Reason : Average value of ac is always defined over half cycle.
9. Assertion : When ac circuit contain resistor only, its power is minimum.
Reason : Power of a circuit is independent of phase angle
10. Assertion : The dc and ac both can be measured by a hot wire instrument.

E
Physics 109
Reason : The hot wire instrument is based on the principle of magnetic effect of current
Type 3 : Multiple choice question (one correct answer) (Questions 11 to 19):-
11. A generator produces a voltage that is given by V  240sin120 t , where t is in seconds. The
frequency and r.m.s. voltage are
(1) 60 Hz and 240 V (2) 19 Hz and 120 V (3) 19 Hz and 170 V (4) 754 Hz and 70 V
12. An inductance of 1 mH a condenser of 10 F and a resistance of 50  are connected in series.
The reactance of inductor and condensers are same. The reactance of either of them will be
(1) 100  (2) 30  (3) 3.2  (4) 10 
13. In a region of uniform magnetic induction B  102 tesla , a circular coil of radius 30 cm and
resistance 2 ohm is rotated about an axis which is perpendicular to the direction of B and which
forms a diameter of the coil. If the coil rotates at 200 rpm the amplitude of the alternating current
induced in the coil is
(1) 42 mA (2) 30 mA (3) 6 mA (4) 200 mA
14. In a series circuit R = 300 , L = 0.9 H, C = 2.0 F and  = 1000 rad/sec. The
impedance of the circuit is
(1) 1300  (2) 900  (3) 500  (4) 400 
 0.4 
15. In a L-R circuit, the value of L is  henry and the value of R is 30 ohm. If in the circuit, an
  
alternating e.m.f. of 200 volt at 50 cycles per sec is connected, the impedance of the circuit and
current will be
(1) 11.4 ,17.5A (2) 30.7 ,6.5A (3) 40.4 ,5A (4) 50 ,4A
16. In a series LCR circuit, resistance R  10 and the impedance Z  20. The phase difference
between the current and the voltage is
(1) 30 o (2) 45o (3) 60 o (4) 90 o
17. A coil of 200 resistance and 1.0 H inductance is connected to an ac source of frequency
200/2 Hz. Phase angle between potential and current will be
(1) 30o (2) 90o (3) 45o (4) 0o
18. A step-down transformer is connected to main supply 200V to operate a 6V, 30W bulb. The
current in primary is
(1) 3 A (2) 1.5 A (3) 0.3 A (4) 0.15 A
19. The output voltage of a transformer connected to 220 volt line is 1100 volt at 1 amp current. Its
efficiency is 100%. The current coming from the line is
(1) 20 A (2) 10 A (3) 11 A (4) 22 A

ANSWER KEY
1. (2) 2. (3) 3. (1) 4. (2) 5. (3) 6. (1) 7. (3) 8. (2)
9. (4) 10. (3) 11. (3) 12. (4) 13. (3) 14. (3) 15. (4) 16. (3)
17. (3) 18. (4) 19. (2)

E
110 Physics

ASSIGNMENT-1

(RMS & Average Value of AC, Reactance, Impedance)

1. An ac source is rated at 220V, 50 Hz. The time taken for voltage to change from its peak value to

zero is

(1) 50 sec (2) 0.02 sec (3) 5 sec (4) 5  103 sec

   
2. V  5sin 100t   and I  4sin 100t  
 6  6

(1) Voltage leads the current by 30 o (2) Current leads the voltage by 30 o

(3) Current leads the voltage by 60 o (4) Voltage leads the current by 60 o

3. A 280 ohm electric bulb is connected to 200V electric line. The peak value of current in the bulb

will be

(1) About one ampere (2) Zero

(3) About two ampere (4) About four ampere

1
4. The frequency for which a 5 F capacitor has a reactance of ohm is given by
1000

100 1000 1
(1) MHz (2) Hz (3) Hz (4) 1000 Hz
  1000

5. The value of the current through an inductance of 1H and of negligible resistance, when

connected through an ac source of 200 V and 50 Hz, is

(1) 0.637 A (2) 1.637 A (3) 2.637A (4) 3.637 A

ASSIGNMENT-2

(Phasor Diagram, LCR series Circuit, Resonance)

6. In a series resonant circuit, the ac voltage across resistance R, inductance L and capacitance C are

5 V, 10 V and 10 V respectively. The ac voltage applied to the circuit will be

(1) 20 V (2) 10 V (3) 5 V (4) 25 V

7. In a LCR circuit having L = 8.0 henry, C = 0.5 F and R = 100 ohm in series. The

resonance frequency in per second is

(1) 600 radian (2) 600 Hz (3) 500 radian (4) 500 Hz

E
Physics 111
8. The variation of the instantaneous current (I) and the instantaneous emf (E) in a circuit is as
shown in fig. Which of the following statements is correct
E I

/2 3/2
O  2 t

(1) The voltage lags behind the current by  / 2


(2) The voltage leads the current by  / 2
(3) The voltage and the current are in phase
(4) The voltage leads the current by 
9. A 10 ohm resistance, 5 mH coil and 10F capacitor are joined in series. When a suitable
frequency alternating current source is joined to this combination, the circuit resonates. If the
resistance is halved, the resonance frequency
(1) Is halved (2) Is doubled
(3) Remains unchanged (4) In quadrupled
10. A circuit has a resistance of 11, an inductive reactance of 25 and a capacitive resistance of
18. It is connected to an ac source of 260V and 50Hz. The current through the circuit (in
amperes) is
(1) 11 (2) 15 (3) 18 (4) 20

ASSIGNMENT-3
(Power in AC circuit, LC oscillation)
11. The power is transmitted from a power house on high voltage ac because
(1) Electric current travels faster at higher volts
(2) It is more economical due to less power wastage
(3) It is difficult to generate power at low voltage
(4) Chances of stealing transmission lines are minimized
12. The power factor of LCR circuit at resonance is
(1) 0.707 (2) 1 (3) Zero (4) 0.5
13. In LCR circuit, the capacitance is changed from C to 4C. For the same resonant frequency, the
inductance should be changed from L to
(1) 2L (2) L/2 (3) L/ 4 (4) 4 L
14. An oscillator circuit consists of an inductance of 0.5mH and a capacitor of 20 F . The resonant
frequency of the circuit is nearly
(1) 15.92 Hz (2) 159.2 Hz (3) 1592 Hz (4) 15910 Hz
15. An inductance of 1 mH a condenser of 10 F and a resistance of 50  are connected in series.
The reactances of inductor and condensers are same. The reactance of either of them will be
(1) 100  (2) 30  (3) 3.2  (4) 10 

E
112 Physics

ASSIGNMENT-4
(AC Generator, Transformer)
16. The number of turns in the coil of an ac generator is 5000 and the area of the coil is 0.25m2 . The

coil is rotated at the rate of 100 cycles/sec in a magnetic field of 0.2 W / m 2 . The peak value of
the emf generated is nearly
(1) 786 kV (2) 440 kV (3) 220 kV (4) 157.1 kV
17. The primary winding of a transformer has 100 turns and its secondary winding has 200 turns.
The primary is connected to an ac supply of 120 V and the current flowing in it is 10 A. The
voltage and the current in the secondary are
(1) 240 V, 5 A (2) 240 V, 10 A (3) 60 V, 20 A (4) 120 V, 20 A
18. A transformer is employed to reduce 220 V to 11 V. The primary draws a current of 5 A and the
secondary 90 A. The efficiency of the transformer is
(1) 20% (2) 40% (3) 70% (4) 90%
19. The efficiency of transformer is very high because
(1) There is no moving part in a transformer (2) It produces very high voltage
(3) It produces very low voltage (4) None of the above
20. A transformer is used to
(1) Change the alternating potential
(2) Change the alternating current
(3) To prevent the power loss in alternating current flow
(4) To increase the power of current source

ANSWER KEY
1. (4) 2. (3) 3. (1) 4. (1) 5. (1) 6. (3) 7. (3) 8. (2)
9. (3) 10. (4) 11. (2) 12. (2) 13. (3) 14. (3) 15. (4) 16. (4)
17. (1) 18. (4) 19. (1) 20. (1)

E
Physics 113

EXERCISE-1
(RMS & Average Value of AC, Reactance, Impedance)
1. If E 0 represents the peak value of the voltage in an ac circuit, the r.m.s. value of the voltage will

be :
E0 E0 E0 E0
(1) (2) (3) (4)
 2  2
2. In an ac circuit I = 100 sin 200 t. The time required for the current to achieve its peak value will
be :
1 1 1 1
(1) sec (2) sec (3) sec (4) sec
100 200 300 400
3. A generator produces a voltage that is given by V  240sin120 t , where t is in seconds. The

frequency and r.m.s. voltage are :


(1) 60 Hz and 240 V (2) 19 Hz and 120 V (3) 19 Hz and 170 V (4) 754 Hz and 70 V
4. If an 8  resistance and 6  reactance are present in an ac series circuit then the impedance of

the circuit will be :

(1) 20 ohm (2) 5 ohm (3) 10 ohm (4) 14 2 ohm


5. The voltage of domestic ac is 220 volt. What does this represent :
(1) Mean voltage (2) Peak voltage
(3) Root mean voltage (4) Root mean square voltage
(Phasor Diagram, LCR series Circuit, Resonance)
6. In the non-resonant circuit, what will be the nature of the circuit for frequencies higher than the
resonant frequency :
(1) Resistive (2) Capacitive (3) Inductive (4) None of the above
7. The vector diagram of current and voltage for a circuit is as shown. The components of the
circuit will be :

45 Erms = 20 V
o

irms = 25
amp
(1) 8/LCR (2) LR (3) LCR or LR (4) None of these

E
114 Physics

8. Power delivered by the source of the circuit becomes maximum, when


2
1  1 
(1) L  C (2) L  (3) L    (4) L  C
C  C 
9. In an LCR series ac circuit, the voltage across each of the components, L, C and R is 50V. The
voltage across the LC combination will be :
(1) 50V (2) 50 2 V (3) 100 V (4) 0 V (zero)
10. In a series LCR circuit, resistance R  10 and the impedance Z  20. The phase difference
between the current and the voltage is :
(1) 30 o (2) 45o (3) 60 o (4) 90 o

(Power in AC circuit, LC oscillation)


11. In an ac circuit, V and I are given by
 
V = 100 sin (100 t) volts, I  100sin 100t   mA .
 3
The power dissipated in circuit is :
(1) 104 watt (2) 10 watt (3) 2.5 watt (4) 5 watt
12. The average power dissipation in a pure capacitance in ac circuit is :
1 1
(1) CV 2 (2) CV 2 (3) CV 2 (4) Zero
2 4
13. For an ac circuit V  15sin t and I  20 cos t the average power consumed in this circuit is :
(1) 300 Watt (2) 150 Watt (3) 75 Watt (4) zero
14. In an ac circuit, the instantaneous values of e.m.f. and current are E = 200(sin 314t)volt and
 
i  sin  314t   ampere. The average power consumed in watt is :
 3
(1) 200 (2) 100 (3) 50 (4) 25

(AC Generator, Transformer)


15. In A.C generator increasing the no. of turns in the coil :
(1) Decreases the Electromotive force (EMF)
(2) Electromotive force (EMF) remains the same
(3) Increases the Electromotive force (EMF)
(4) Electromotive force (EMF) becomes zero
16. In alternative current generator, AC current reverses its direction :
(1) 10 times per second (2) 20 times per second
(3) 60 times per second (4) 50 times per second

E
Physics 115
17. The direction of induced emf or current is indicated by
(1) Fleming’s left-hand rule
(2) Fleming’s right-hand rule
(3) Faraday’s law of electromagnetic induction
(4) None of these
18. A transformer is based on the principle of :
(1) Mutual Induction (2) Self-Induction (3) Ampere’s Law (4) None of these
19. Transformer eddy current losses can be reduced by :
(1) Using copper wire in winding
(2) Using laminated core
(3) Using step down transformer
(4) Increasing the number of turns in secondary coil
20. Transformation ratio in the step –up transformer is
(1) One
(2) Greater than one
(3) Less than one
(4) The ratio greater or less than one depends on the other factor

ANSWER KEY
1. (4) 2. (4) 3. (3) 4. (3) 5. (4) 6. (2) 7. (3) 8. (2)
9. (4) 10. (3) 11. (3) 12. (2) 13. (4) 14. (3) 15. (3) 16. (4)
17. (2) 18. (1) 19. (2) 20. (2)

E
116 Physics

CASE STUDY BASED QUESTIONS

(Transformer)
A transformer is essentially an a.c. device. It cannot work on d.c. It changes alternating voltages
or currents. It does not affect the frequency of a.c. It is based on the phenomenon of mutual
induction. A transformer essentially consists of two coils of insulated copper wire having
different number of turns and wound on the same soft iron core.
The number of turns in the primary and secondary coils of an ideal transformer are 2000 and
50 respectively. The primary coil is connected to a main supply of 120 V and secondary coil is
connected to a bulb of resistance 0.6 .
1. The value of voltage across the secondary coil is :
(1) 5 V (2) 2 V (3) 3 V (4) 10 V
2. The value of current in the bulb is :
(1) 7 A (2) 15 A (3) 3 A (4) 5 A
3. The value of current in primary coil is :
(1) 0.125 A (2) 2.52 A (3) 1.51 A (4) 3.52 A
4. Power in primary coil is :
(1) 20 W (2) 5 W (3) 10 W (4) 15 W
5. Power in secondary coil is :
(1) 15 W (2) 20 W (3) 7 W (4) 8 W

ANSWER KEY
1. (3) 2. (4) 3. (1) 4. (4) 5. (1)

E
Physics 117
ASSERTION AND REASON QUESTIONS
Directions (Q. Nos. 1-5)

In the following questions an Assertion (A) is followed by a corresponding Reason (R). Use the

following keys to choose the appropriate answer.

(1) Both (A) and (R) are true, (R) is the correct explanation of (A).

(2) Both (A) and (R) are true, (R) is not the correct explanation of (A).

(3) (A) is true; (R) is false.

(4) (A) is false; (R) is false.

1. Assertion : The alternating current lags behind the emf. by a phase angle of /2, when A.C.

flows through an inductor.

Reason : The inductive reactance increases as the frequency of A.C. source decreases.

2. Assertion : Capacitor serves as a block for D.C. and offers an easy path to A.C.

Reason : Capacitive reactance is inversely proportional to frequency.

3. Assertion : When capacitive reactance is smaller than the inductive reactance in LCR series

circuit, e.m.f. leads the current.

Reason : The phase angle is the angle between the alternating e.m.f. and alternating current of

the circuit.

4. Assertion : In series LCR circuit phase difference between current and voltage is never zero.

Reason : Voltage and current are in phase.

5. Assertion : Average power loss in series LC circuit is always zero.

Reason : Average value of voltage and current in AC is zero.

ANSWER KEY
1. (3) 2. (1) 3. (2) 4. (4) 5. (2)

E
118 Physics

EXERCISE-2

 
1. If a current I given by I0 sin   t   flows in an ac circuit across which an ac potential of
 2
E  E0 sin  t has been applied, then the power consumption P in the circuit will be

E0 I0 E0 I0
(1) P  (2) P  2E 0 I 0 (3) P  (4) P = 0
2 2
2. The r.m.s. value of an ac of 50 Hz is 10 amp. The time taken by the alternating current in
reaching from zero to maximum value and the peak value of current will be
(1) 2  10–2 sec and 14.14 amp (2) 1  10–2 sec and 7.07 amp
(3) 5  10–3 sec and 7.07 amp (4) 5  10–3 sec and 14.14 amp
3. The current 'i' in an inductance coil varies with time 't' according to following graph

t
(0, 0)

Which one of the following plots shows the variations of voltage in the coil
V V

(1) (2)
t (0, 0) t
(0, 0)

V
V

(3) (4)
(0, 0) t
(0, 0) t

4. A bulb is connected first with dc and then ac of same voltage then it will shine brightly with
(1) AC (2) DC
(3) Brightness will be in ratio 1/1.4 (4) Equally with both
5. In the circuit given below, what will be the reading of the voltmeter :
V 100V 100V

200V, 100 Hz

(1) 300 V (2) 900 V (3) 200 V (4) 400 V

E
Physics 119
6. For high frequency, a capacitor offers
(1) More reactance (2) Less reactance (3) Zero reactance (4) Infinite reactance
7. An alternating e.m.f. of angular frequency  is applied across an inductance. The instantaneous
power developed in the circuit has an angular frequency
 
(1) (2) (3)  (4) 2
4 2
8. In an ac circuit, the current lags behind the voltage by  / 3 . The components in the circuit are
(1) R and L (2) R and C (3) L and C (4) Only R
9. Same current is flowing in two alternating circuits. The first circuit contains only inductance and
the other contains only a capacitor. If the frequency of the e.m.f. of ac is increased, the effect on
the value of the current will be
(1) Increases in the first circuit and decreases in the other
(2) Increases in both the circuits
(3) Decreases in both the circuits
(4) Decreases in the first circuit and increases in the other
10. The figure shows variation of R, XL and XC with frequency f in a series L, C, R circuit. Then for
what frequency point, the circuit is inductive

XC XL

f
AB C

(1) A (2) B (3) C (4) All points

ASSERTION & REASON QUESTIONS (11 to 20)


In the following questions an Assertion (A) is followed by a corresponding Reason (R). Use the
following keys to choose the appropriate answer.
(1) Both (A) and (R) are true, (R) is the correct explanation of (A).
(2) Both (A) and (R) are true, (R) is not the correct explanation of (A).
(3) (A) is true; (R) is false.
(4) (A) is false; (R) is false.
11. Assertion : An electric lamp connected in series with a variable capacitor and A.C. source, its
brightness increases with increase in capacitance.

E
120 Physics

Reason : Capacitive reactance decreases with increase in capacitance of capacitor.


12. Assertion : Soft iron is used as a core of transformer.
Reason : Area of hysteresis loop for soft iron is small.
13. Assertion : We use a thick wire in the secondary coil of a step down transformer to reduce the
production of heat.
Reason : Laminated core used to reduce eddy current losses.
14. Assertion : An inductance and a resistance are connected in series with an A.C. circuit. In this
circuit the current across the resistance lags behind potential difference across the inductance by
an angle /2.
Reason : In L-R circuit voltage lags the current by phase angle which depends on the value of
inductance and resistance both.
15. Assertion : A bulb connected in series with a solenoid is connected to A.C. source. If a soft iron
core is introduced in the solenoid, the bulb will glow brighter.
Reason : On introducing soft iron core in the solenoid, the inductance decreases.
16. Assertion : An alternating current does not show any magnetic effect.
Reason : Alternating current does not vary with time.
17. Assertion : A capacitor of suitable capacitance can be used in an ac circuit in place of the choke
coil.
Reason : A capacitor blocks dc and allows ac only.
18. Assertion : The divisions are equally marked on the scale of ac ammeter.
Reason : Heat produced is directly proportional to the current.
19. Assertion : AC is more dangerous than DC.
Reason : Peak value of AC is more that of the DC for the same voltage.
20. Assertion : In series LCR circuit resonance can take place.
Reason : Resonance takes place if inductance and capacitive reactance’s are equal and opposite.

ANSWER KEY
1. (4) 2. (4) 3. (2) 4. (4) 5. (3) 6. (2) 7. (4) 8. (1)
9. (4) 10. (3) 11. (1) 12. (1) 13. (2) 14. (4) 15. (4) 16. (4)
17. (2) 18. (4) 19. (1) 20. (1)

You might also like